July 2016
New York State
Bar Examination
MEE & MPT Questions
© 2016
National Conference of Bar Examiners
© 2016
National Conference of Bar Examiners
These materials are copyrighted by the NCBE and are reprinted with the permission of NCBE.
These materials are for personal use only and may not be reproduced or distributed in any way.
1
MEE 1
Two siblings, a brother and a sister, decided to start a bike shop with their cousin. They
filed a certificate of organization to form a limited liability company. The brother and the
sister paid for their LLC member interests by each contributing $100,000 in cash to the
LLC. Their cousin paid for his LLC member interest by conveying to the LLC five acres
of farmland valued at $100,000; the LLC then recorded the deed.
Neither the certificate of organization nor the members’ operating agreement specifies
whether the LLC is member-managed or manager-managed. However, the operating
agreement provides that the LLC’s farmland may not be sold without the approval of all
three members.
Following formation of the LLC, the company rented a storefront commercial space for
the bike shop and opened for business.
Three months ago, purporting to act on behalf of the LLC, the brother entered into a
written and signed contract to purchase 100 bike tires for $6,000 from a tire
manufacturer. When the tires were delivered, the sister said that they were too expensive
and told her brother to return the tires. The brother was surprised by his sister’s objection
because twice before he had purchased tires for the LLC at the same price from this
manufacturer, and neither his sister nor their cousin had objected. The brother refused to
return the tires, pointing out that the tires “are perfect for the bikes we sell.” The sister
responded, “Well, pay the bill with your own money; you bought them without my
permission.” The brother responded, “No way. I bought these for the store, I didn’t need
your permission, and the company will pay for them.” To date, however, the $6,000 has
not been paid.
One month ago, purporting to act on behalf of the LLC, the cousin sold the LLC’s
farmland to a third-party buyer. The buyer paid $120,000, which was well above the
land’s fair market value. Only after the cousin deposited the sale proceeds into the LLC
bank account did the brother and sister learn of the sale. Both of them objected.
One week ago, the brother wrote in an email to his sister, “I want out of our business. I
don’t want to have anything to do with the bike shop anymore. Please send me a check
for my share.”
© 2016
National Conference of Bar Examiners
These materials are copyrighted by the NCBE and are reprinted with the permission of NCBE.
These materials are for personal use only and may not be reproduced or distributed in any way.
2
1. What type of LLC was created—member-managed or manager-managed?
Explain.
2. Is the LLC bound under the tire contract? Explain.
3. Is the LLC bound by the sale of the farmland? Explain.
4. What is the legal effect of the brother’s email? Explain.
MEE 2
A defendant was tried before a jury for a robbery that had occurred at Jo-Jo’s Bar on
November 30. At trial, the prosecutor called the police officer who had investigated the
crime. Over defense counsel’s objection, the officer testified as follows:
Officer: I arrived at the defendant’s home on the morning of December 1, the day
after the robbery. He invited me inside, and I asked him, “Did you rob Jo-Jo’s Bar
last night?” The defendant immediately started crying. I decided to take him to the
station. Before we left for the station, I read him Miranda warnings, and he said,
“Get me a lawyer,” so I stopped talking to him.
Prosecutor: Did the defendant say anything to you at the station?
Officer: I think he did, but I don’t remember exactly what he said.
Immediately after this testimony, the prosecutor showed the officer a handwritten
document. The officer identified the document as notes she had made on December 2
concerning her interaction with the defendant on December 1. The prosecutor provided a
copy of the document to defense counsel. The document, which was dated December 2,
stated in its entirety:
The defendant burst into tears when asked if he had committed the robbery. He
then received and invoked Miranda rights. I stopped the interrogation and didn’t
ask him any more questions, but as soon as we arrived at the station the defendant
said, “I want to make a deal; I think I can help you.” I reread Miranda warnings,
and this time the defendant waived his rights and said,I have some information
© 2016
National Conference of Bar Examiners
These materials are copyrighted by the NCBE and are reprinted with the permission of NCBE.
These materials are for personal use only and may not be reproduced or distributed in any way.
3
that can really help you with this case.” When I asked him how he could help, the
defendant said, “Forget itI want my lawyer.” When the defendant’s lawyer
arrived 30 minutes later, the defendant was released.
The officer then testified as follows:
Prosecutor: After reviewing your notes, do you remember the events of December
1?
Officer: No, but I do remember making these notes the day after I spoke with the
defendant. At that time, I remembered the conversation clearly, and I was careful
to write it down accurately.
Over defense counsel’s objection, the officer was permitted to read the document to the
jury. The prosecutor also asked that the notes be received as an exhibit, and the court
granted that request, again over defense counsel’s objection. The testimony then
continued:
Prosecutor: Did you speak to the defendant any time after December 1?
Officer: Following my discovery of additional evidence implicating the defendant
in the robbery, I arrested him on December 20. Again, I read the defendant his
Miranda rights. The defendant said that he would waive his Miranda rights. I then
asked him if he was involved in the robbery of Jo-Jo’s Bar, and he said, “I was
there on November 30 and saw the robbery, but I had nothing to do with it.”
Defense counsel objected to the admission of this testimony as well. The court overruled
the objection.
The defendant’s trial for robbery was held in a jurisdiction that has adopted all of the
Federal Rules of Evidence.
Were the following decisions by the trial court proper?
1. Admitting the officer’s testimony that the defendant started crying. Explain.
2. Permitting the officer to read her handwritten notes to the jury. Explain.
3. Admitting the officer’s handwritten notes into evidence as an exhibit. Explain.
© 2016
National Conference of Bar Examiners
These materials are copyrighted by the NCBE and are reprinted with the permission of NCBE.
These materials are for personal use only and may not be reproduced or distributed in any way.
4
4. Admitting the officer’s testimony recounting the defendant’s statement, “I have
some information that can really help you with this case.” Explain.
5. Admitting the officer’s testimony recounting the defendant’s statement, “I was
there on November 30 and saw the robbery, but I had nothing to do with it.”
Explain.
MEE 3
Six months ago, a man visited his family physician, a general practitioner, for a routine
examination. Based on blood tests, the physician told the man that his cholesterol level
was somewhat elevated. The physician offered to prescribe a drug that lowers cholesterol,
but the man stated that he did not want to start taking drugs because he preferred to try
dietary change and “natural remedies” first. The physician told the man that natural
remedies are not as reliable as prescription drugs and urged the man to come back in
three months for another blood test. The physician also told the man about a recent
research report showing that an herbal tea made from a particular herb can reduce
cholesterol levels.
The man purchased the herbal tea at a health-food store and began to drink it. The man
also began a cholesterol-lowering diet.
Three months ago, the man returned to his physician and underwent another blood test;
the test showed that the man’s cholesterol level had declined considerably. However, the
test also showed that the man had an elevated white blood cell count. The man’s test
results were consistent with several different infections and some types of cancer. Over
the next two weeks, the physician had the man undergo more tests. These tests showed
that the man’s liver was inflamed but did not reveal the reason. The physician then
referred the man to a medical specialist who had expertise in liver diseases. In the
meantime, the man continued to drink the herbal tea.
Two weeks ago, just before the man’s scheduled consultation with the specialist, the man
heard a news bulletin announcing that government investigators had found that the type
of herbal tea that the man had been drinking was contaminated with a highly toxic
pesticide. The investigation took place after liver specialists at a major medical center
realized that several patients with inflamed livers and elevated white blood cell counts,
© 2016
National Conference of Bar Examiners
These materials are copyrighted by the NCBE and are reprinted with the permission of NCBE.
These materials are for personal use only and may not be reproduced or distributed in any way.
5
like the man, were all drinking the same type of herbal tea and the specialists reported
this fact to the local health department.
All commercially grown herbs used for this tea come from Country X, and are tested for
pesticide residues at harvest by exporters that sell the herb in bulk to the five U.S.
companies that process, package, and sell the herbal tea to retailers. U.S. investigators
believe that the pesticide contamination occurred in one or more export warehouses in
Country X where bulk herbs are briefly stored before sale by exporters, but they cannot
determine how the contamination occurred or what bulk shipments were sent to the five
U.S. companies. The companies that purchase the bulk herbs do not have any control
over these warehouses, and there have been no prior incidents of pesticide contamination.
The investigators have concluded that the U.S. companies that process, package, and sell
the herbal tea were not negligent in failing to discover the contamination.
Packages of tea sold by different companies varied substantially in pesticide
concentration and toxicity, and some packages had no contaminants. Further
investigation has established that the levels of contamination and toxicity in the herbal tea
marketed by the five different U.S. companies were not consistent.
The man purchased all his herbal tea from the same health-food store. The man is sure
that he purchased several different brands of the herbal tea at the store, but he cannot
establish which brands. The store sells all five brands of the herbal tea currently marketed
in the United States.
The man has suffered permanent liver damage and has sued to recover damages for his
injuries. It is undisputed that the man’s liver damage was caused by his herbal tea
consumption. The man’s action is not preempted by any federal statute or regulation.
1. Is the physician liable to the man under tort law? Explain.
2. Are any or all of the five U.S. companies that processed, packaged, and sold the
herbal tea to the health-food store liable to the man under tort law? Explain.
3. Is the health-food store liable to the man under tort law? Explain.
© 2016
National Conference of Bar Examiners
These materials are copyrighted by the NCBE and are reprinted with the permission of NCBE.
These materials are for personal use only and may not be reproduced or distributed in any way.
6
MEE 4
Two years ago, PT Treatment Inc. (PTT), incorporated in State A, decided to build a new
$90 million proton-therapy cancer treatment center in State A. The total cost to PTT for
purchasing the land and constructing the building to house the treatment facility was $30
million. PTT financed the purchase and construction with $10 million of its own money
and $20 million that it borrowed from Bank. To secure its obligation to Bank, PTT
granted Bank a mortgage on the land and all structures erected on the land. The mortgage
was properly recorded in the county real estate records office, but it was not identified as
a construction mortgage.
Two months after the mortgage was recorded, PTT finalized an agreement for the
purchase of proton-therapy equipment from Ion Medical Systems (Ion) for $60 million.
PTT made a down payment of $14 million and signed a purchase agreement promising to
pay the remaining $46 million in semi-annual payments over a 10-year period. The
purchase agreement provided that Ion has a security interest in the proton-therapy
equipment to secure PTT’s obligation to pay the remaining purchase price. On the same
day, Ion filed a properly completed financing statement with the office of the Secretary of
State of State A (the central statewide filing office designated by statute), listing “PT
Treatment Inc.” as debtor and indicating the proton-therapy equipment as collateral.
Shortly thereafter, Ion delivered the equipment to PTT and PTT’s employees installed it.
The equipment was attached to the building in such a manner that, under State A law, it is
considered a fixture and an interest in the equipment exists in favor of anyone with an
interest in the building.
The new PTT Cancer Treatment Center opened for business last year. Unfortunately, it
has not been an economic success. For a short period, PTT contracted with State A
Oncology Associates (Oncology) for the latter’s use of the proton-therapy equipment
pursuant to a lease agreement, but Oncology failed to pay the agreed fee for the use of the
equipment, so PTT terminated that arrangement. To date, PTT has been unsuccessful in
its efforts to collect the amounts that Oncology still owes it. PTT’s own doctors and
technicians have not attracted enough business to fully utilize the cancer treatment center
or generate sufficient billings to meet PTT’s financial obligations. PTT currently owes
Ion more than $30 million and is in default under the security agreement. Ion is
concerned that PTT will soon declare bankruptcy.
© 2016
National Conference of Bar Examiners
These materials are copyrighted by the NCBE and are reprinted with the permission of NCBE.
These materials are for personal use only and may not be reproduced or distributed in any way.
7
In a few days, Ion will be sending a technician to the PTT facility to perform regular
maintenance on the equipment. Ion is considering instructing the technician to complete
the maintenance and then disable the equipment so that it cannot be used by PTT until
PTT pays what it owes.
1. In view of PTT’s default, if Ion disables the proton-therapy equipment, will it
incur any liability to PTT? Explain.
2. If PTT does not pay its debts to either Bank or Ion, which of them has a superior
claim to the proton-therapy equipment? Explain.
3. Does Ion have an enforceable and perfected security interest in any of PTT’s
assets other than the proton-therapy equipment? Explain.
MEE 5
A homeowner and his neighbor live in houses that were built at the same time. The two
houses have identical exteriors and are next to each other. The homeowner and his
neighbor have not painted their houses in a long time, and the exterior paint on both
houses is cracked and peeling. A retiree, who lives across the street from the homeowner
and the neighbor, has complained to both of them that the peeling paint on their houses
reduces property values in the neighborhood.
Last week, the homeowner contacted a professional housepainter. After some discussion,
the painter and the homeowner entered into a written contract, signed by both of them,
pursuant to which the painter agreed to paint the homeowner’s house within 14 days and
the homeowner agreed to pay the painter $6,000 no later than three days after completion
of painting. The price was advantageous for the homeowner because, to paint a house of
that size, most professional housepainters would have charged at least $8,000.
The day after the homeowner entered into the contract with the painter, he told his
neighbor about the great deal he had made. The neighbor then stated that her parents
wanted to come to town for a short visit the following month, but that she was reluctant
to invite them. “This would be the first time my parents would see my house, but I can’t
invite them to my house with its peeling paint; I’d be too embarrassed. I’d paint the house
now, but I can’t afford the going rate for a good paint job.”
© 2016
National Conference of Bar Examiners
These materials are copyrighted by the NCBE and are reprinted with the permission of NCBE.
These materials are for personal use only and may not be reproduced or distributed in any way.
8
The homeowner, who was facing cash-flow problems of his own, decided to offer the
neighbor a deal that would help them both. The homeowner said that, for $500, the
homeowner would allow the neighbor to take over the homeowner’s rights under the
contract. The homeowner said, “You’ll pay me $500 and take the contract from me; the
painter will paint your house instead of mine, and when he’s done, you’ll pay him the
$6,000.” The neighbor happily agreed to this idea.
The following day, the neighbor paid the homeowner $500 and the homeowner said to
her, “The paint deal is now yours.” The neighbor then invited her parents for the visit that
had been discussed. The neighbor also remembered how annoyed the retiree had been
about the condition of her house. Accordingly, she called the retiree and told him about
the plans to have her house painted. The retiree responded that it was “about time.”
Later that day, the homeowner and the neighbor told the painter about the deal pursuant
to which the neighbor had taken over the contract from the homeowner. The painter was
unhappy with the news and stated, “You can’t change my deal without my consent. I will
honor my commitment to paint the house I promised to paint, but I won’t paint someone
else’s house.”
There is no difference in magnitude or difficulty between the work required to paint the
homeowner’s house and the work required to paint the neighbor’s house.
1. If the painter refuses to paint the neighbor’s house, would the neighbor succeed in
a breach of contract action against the painter? Explain.
2. Assuming that the neighbor would succeed in the breach of contract action against
the painter, would the retiree succeed in a breach of contract action? Explain.
3. If the painter paints the neighbor’s house and the neighbor does not pay the $6,000
contract price, would the painter succeed in a contract claim against the neighbor?
Against the homeowner? Explain.
© 2016
National Conference of Bar Examiners
These materials are copyrighted by the NCBE and are reprinted with the permission of NCBE.
These materials are for personal use only and may not be reproduced or distributed in any way.
9
MEE 6
A woman and a man have both lived their entire lives in State A. The man once went to a
gun show in State B where he bought a gun. Otherwise, neither the woman nor the man
had ever left State A until the following events occurred.
The woman and the man went hunting for wild turkey at a State A game preserve. The
man was carrying the gun he had purchased in State B. The man had permanently
disabled the gun’s safety features to be able to react more quickly to a turkey sighting.
The man dropped the gun and it accidentally fired, inflicting a serious chest wound on the
woman. The woman was immediately flown to a hospital in neighboring State C, where
she underwent surgery.
One week after the shooting accident, the man traveled to State C for business and took
the opportunity to visit the woman in the hospital. During the visit, the woman’s attorney
handed the man the summons and complaint in a suit the woman had initiated against the
man in the United States District Court for the District of State C. Two days later, the
woman was released from the hospital and returned home to State A where she spent
weeks recovering.
The woman’s complaint alleges separate claims against the man: 1) a state-law
negligence claim and 2) a federal claim under the Federal Gun Safety Act (Safety Act).
The Safety Act provides a cause of action for individuals harmed by gun owners who
alter the safety features of a gun that has traveled in interstate commerce. The Safety Act
caps damages at $100,000 per incident, but does not preempt state causes of action. The
woman’s complaint seeks damages of $100,000 on the Safety Act claim and $120,000 on
the state-law negligence claim. Both sets of damages are sought as compensation for the
physical suffering the woman experienced and the medical costs the woman incurred as a
result of the shooting.
The man has moved to dismiss the complaint, asserting (a) lack of personal jurisdiction,
(b) lack of subject-matter jurisdiction, and (c) improper venue. State C’s jurisdictional
statutes provide that state courts may exercise personal jurisdiction to the limits allowed
by the United States Constitution.”
With respect to each asserted basis for dismissal, should the man’s motion to dismiss be
granted? Explain.
© 2016
National Conference of Bar Examiners
These materials are copyrighted by the NCBE and are reprinted with the permission of NCBE.
These materials are for personal use only and may not be reproduced or distributed in any way.
10
MPT 1 – In re Whirley (Synopsis)
In this performance test item, examinees are associates at a law firm representing Barbara
Whirley. In January, Whirley began renting a house from Sean Spears. In the last few
months, Whirley has had some problems with the housea leaking toilet, a broken
automatic sprinkler system, and a defective sliding door in the guest bedroom, which has
allowed water to enter and cause the carpet to get mildewed and moldy. In addition,
Whirley’s dog has chewed on the baseboard in the laundry room. She has told Spears
about the problems with the toilet, sprinkler system, and door, but he has failed to make
any repairs. Whirley seeks the firm’s advice regarding her options as a tenant.
Examinees’ task is to draft an objective memorandum identifying the options Whirley has
under Franklin’s landlord/tenant law to address each condition in the house and
recommending which options are best, keeping in mind that Whirley prefers to continue
living in the house. The File contains the instructional memorandum from the supervising
attorney, a summary of the client interview, the lease agreement, an email exchange
between Whirley and Spears, and a repair estimate. The Library contains excerpts from
the Franklin Civil Code and two Franklin cases that discuss what conditions may
constitute breaches of the warranty of tenantability and the tenant’s potential remedies.
MPT 2 Nash v. Franklin Department of Revenue (Synopsis)
Examinees’ law firm represents Joseph and Ellen Nash, a married couple who own land
in Knox Hollow, Franklin, on which they raise Christmas trees for sale. While initially
selling the trees was a casual endeavor, five years ago they made it a commercial tree-
farming operation and began claiming tax deductions for expenses from a trade or
business. The Franklin Department of Revenue recently reviewed the Nashes’ income tax
returns and has disallowed the Nashes’ deductions for the last five years’ farm expenses,
as well as their claim for a home-office deduction. The Nashes appealed the new tax
assessment, and the firm represented the Nashes at a hearing before the Franklin Tax
Court. Examinees’ task is to draft the legal argument for the post-hearing brief requested
by the Tax Court, making the case that the Nashes are entitled to the full deductions that
they claimed under Franklin law. [Franklin law uses the federal Internal Revenue Code
and Regulations to calculate Franklin tax liability.] The File contains the instructional
memorandum, the firm’s guidelines for drafting briefs, the decision by the Franklin
Department of Revenue, and a transcript of Joseph Nash’s testimony before the Franklin
Tax Court. The Library contains relevant excerpts from Internal Revenue Code §§ 162,
© 2016
National Conference of Bar Examiners
These materials are copyrighted by the NCBE and are reprinted with the permission of NCBE.
These materials are for personal use only and may not be reproduced or distributed in any way.
11
183, and 280A, and Internal Revenue regulations (26 C.F.R. § 1.1832). The Library also
contains two cases from the Franklin Tax Court addressing what it means for a taxpayer
to be engaged in an “activity for profit,” and the standard for whether a taxpayer has used
a portion of his or her home “exclusively” as the principal place of business of a business.
July 2016
New York State
Bar Examination
Sample Answers
JULY 2016 NEW YORK STATE BAR EXAMINATION
SAMPLE CANDIDATE ANSWERS
The following are sample candidate answers that received scores superior to the average
scale score awarded for the relevant essay. They have been reprinted without change,
except for minor editing. These essays should not be viewed as "model" answers, and
they do not, in all respects, accurately reflect New York State law and/or its application
to the facts. These answers are intended to demonstrate the general length and quality of
responses that earned above average scores on the indicated administration of the bar
examination. These answers are not intended to be used as a means of learning the law
tested on the examination, and their use for such a purpose is strongly discouraged.
1
ANSWER TO MEE 1
1. The LLC is Member-Managed.
The issue is whether Brother Sister and Cousin are running the LLC as members,
or if they are acting as managers. Where an LLC certificate of organization and operating
agreement are silent as to whether the LLC is managed by members or managers, the
default is that the members are managing the LLC because otherwise they would need to
appoint managers. An LLC is a business organization that takes qualities of a partnership
for tax purposes and has limited liability like corporations. Members are designated by
their contributions in return for an interest in the LLC. Contributions can either be in
cash, property, or past-services.
Here the members are Brother Sister and Cousin, because Brother and Sister
contributed each $100,000 cash, and Cousin contributed five acres of farmland valued at
$100,000. Thus in return they were members of the LLC, and as such ran the company.
Here the Operating Agreement and Articles of Organization are silent as to whether the
LLC is member-managed or manager-managed, thus they did not designate themselves or
others as managers, and are now running a Member-Managed LLC.
2. The LLC is bound under the tire contract.
The issue is whether Brother had the authority to enter into and bind the LLC to
the tire contract. Members of an LLC owe each other and the LLC fiduciary duties
similar to that of a partnership, and thus owe each other and the LLC the utmost duty of
care. With that they also have similar powers of partners in a partnership, in which
partners have the authority to enter into business contracts on behalf of the company.
Authority to enter into contracts can be actual express authority, actual implied authority,
or apparent authority. Actual express authority is that authority that is expressly granted
in a writing or oral. Actual Implied Authority is authority that ordinary runs with people
with the same title or position. Apparent authority is that which the company holds
out/cloaks its agents as their own with the authority to act, and a third party relying on the
cloaking of the company and enter into the contract.
Here brother, Sister, and Cousin have the actual implied authority as members of
an LLC to enter into ordinary business contracts on behalf of the LLC because while the
Operating Agreement and Articles of Organization are silent as to whether they are
members or managers of the LLC, they will have the implied authority as members of an
LLC to enter into ordinary business contracts. The contract in question here is for tires
from a manufacturer, and the LLC runs a bike shop, thus it would be in the ordinary
course of business to order tires for the bike shop. Because this contract falls into the
ordinary course of business then Brother had the actual implied authority as a member to
enter into this contract. Even in the event that this was not actual implied authority, it
2
would arguably be apparent authority because the manufacturer and brother have entered
into two other contracts for tires without any objection from sister and cousin/the LLC,
thus the LLC held out Brother as able to enter into the contract, by taking the benefit of
the contract. Then the manufacturer relied on this apparent authority and entered into a
third contract with brother.
Thus brother has the actual implied authority, and if not, then apparent authority to
enter into the tire contract and bind the LLC.
3. The LLC is NOT bound by the sale of the farmland.
The issue is whether Cousin had the authority to sell the land on his own. LLC
property is not transferable without the written consent of all of the members. LLC
property includes anything that the LLC paid for with its own money. Additionally,
contributions become LLC property when they are transferred over to the LLC in return
for an interest in the LLC. However where there is apparent authority, which is when the
company holds out/cloaks its agents as their own with the authority to act on behalf of the
LLC, and a third party relying on the cloaking of the company and enter into the contract.
Here, Cousin conveyed to the LLC Five acres of farmland valued at $100,000 in
return for his membership interest in the LLC. The farmland then became LLC property
when they completed the transfer and recorded the deed. At that point the land is the
LLC's and is not able to be transferred without the written consent of all of the members.
A land sale contract is also not in the ordinary course of business of the LLC since it is a
bike shop, thus it would not be considered implied authority. It may be argued that cousin
had the apparent authority to enter into the land sale contract since he was purporting to
act on behalf of the LLC, however the LLC is not cloaking Cousin with the authority to
enter into land sale K's since this is a bike shop, thus a third party should not reasonably
rely on an agent having that authority. Thus because it would be unreasonable for a third
party to believe that Cousin could enter into a land sale contract for a bike shop, there
was no apparent authority. The contract was outside the scope of Cousin's authority, was
LLC property, and thus not transferrable without the written consent of all of the
members.
4. The legal effect of Brother's email is that he is a dissociating member.
The issue is whether Brother is unequivocally giving up his interest in the
membership. A member will be dissociating when they unequivocally express their intent
to end their interest in the LLC. A dissociating member will remain liable on the debts
and obligations of the LLC until notice is given to all creditors of the dissociating
member, OR until 90 days after filing with the secretary of state the intent to dissociate.
Here brother is unequivocally expressing his intent to end his interest in the LLC because
he has clearly stated that he wants out of the business, does not want to have anything to
3
do with the bike shop anymore and wants to be paid out for his share of the LLC. This is
clearly expressed intent of dissociation, thus he will remain liable on the debts and
obligations of the LLC until notice is given to all creditors of his dissociation or until
after 90 days from the filing with the secretary of state.
ANSWER TO MEE 1
The first issue is whether the LLC is member-managed or manager-managed in
the absence of a specification stated in the certificate of organization or operating
agreement. The rule is that unless stated otherwise, LLCs will be presumed to be
managed by members. In this case, there was never an appointment of managers to
manage the LLC, so by default the members would manage the LLC.
The second issue is when an LLC will be bound on a contract made in the ordinary
course of business. Assuming the LLC is member-managed, the members will operate as
agents of the LLC, and can act to bind the LLC in contracts in the ordinary course of
business unless the operating agreement or certificate of organization specify otherwise.
A principal is bound on the authorized contracts of its agents. Authorization can be
express actual authority, implied actual authority, apparent authority, or based on
ratification. In this case, the brother's contract to purchase 100 bike tires for $6000 is
within the ordinary course of business, because the LLC is a bike shop. Thus, he has
actual authority as an agent to bind the LLC to this contract. This can be implied from the
fact that he has entered into the same contract with the same manufacturer twice before,
and neither his sister nor cousin objected. Therefore, based on his prior dealings with the
LLC, he has actual authority to enter into this contract. He would also have apparent
authority to enter into this contract. Apparent authority comes from the principal's
cloaking a person with the appearance of authority and a third party reasonably relying on
that. Here, the LLC cloaked the brother with the appearance of authority by having him
enter into a contract with a manufacturer that he had entered into twice before, and the
manufacturer reasonably relied on it in making the contract. Therefore, the LLC is bound
by the contract, and will have to pay for the tires.
The third issue is whether the LLC will be bound on a contract made outside the
ordinary course of business. The sale of farmland is outside the ordinary course of
business for a bike shop. Therefore, the members must agree to enter into such a contract.
This would normally require majority consent. Here, the LLC's operating agreement
expressly states that the LLC's farmland may not be sold without approval of all three
members, so sale of the farmland would require unanimous consent. Thus, the cousin's
sale of the farmland was not done with actual authority, because the brother and the sister
did not approve of it, and they objected once they found out. The sale was to a third party
buyer, and the facts indicate that the cousin entered into the sale purporting to act on the
4
LLC's behalf. However, nothing the LLC did would have given the cousin the appearance
of authority to sell the farmland from the third-party's perspective, so there is no apparent
authority here. If there were apparent authority (if the third-party buyer reasonably relied
on the LLC's purported cloaking the cousin with authority by virtue of the fact that he
was an LLC member), then the LLC would be bound by the sale of the farmland.
The fourth issue is what effect of a dissociating member will be on the LLC. The
brother's email has the effect of causing his dissociation. When a member dissociates, the
LLC can choose to continue and buy out the dissociating member. The LLC may
terminate upon his dissociation, in which case the members would have to wind up the
business.
ANSWER TO MEE 2
1. Admitting the officer's testimony that the defendant started crying was proper.
At issue is whether this is a statement and therefore whether it is subject to possible
suppression on 5th Amendment grounds or hearsay grounds. Hearsay is defined as an
out of court statement offered for the truth of the matter asserted. Under the Federal
Rules of Evidence, all hearsay must be excluded unless it falls within an exception found
in FRE 803. However, excluded from the definition of hearsay is a statement by a party
opponent (a statement made by the party not seeking to introduce it). Here, if this were
found to be a statement, the defendant made it and therefore its admission would not be
barred on hearsay grounds. However, bodily responses like crying are not likely to be
deemed statements anyway as crying under ordinary circumstances would not be meant
to convey an idea or express something but rather a reaction that is physical and can be
observed by anyone within sight.
The 5th Amendment protects defendants from self-incrimination by excluding
from evidence statements made without proper Miranda warnings while subject to
custodial interrogation. However, this only applies to statements. As stated above, the
involuntary response of crying is unlikely to be deemed a statement and therefore its
introduction cannot violate the defendant's 5th Amendment rights.
Even if the crying was found to be a statement, it was still properly admitted. At
issue is whether the defendant was in custody when he began crying. Miranda warnings
warn a suspect or defendant that anything they say can be used against them in a court
and that they have a right to an attorney and that one will be provided for them if they
cannot afford it. Miranda warnings must be given prior to custodial interrogation for the
resulting statements to be admissible in court. For a statement to be suppressed because
it was Miranda deficient, it must have been made as a result of custodial interrogation. A
person will be found to be in custody if a reasonable innocent person in their situation
5
would not feel free to leave. Here, the defendant invited the officer into his home.
Unless the officer was there pursuant to a warrant or entered with some sort of force or
intimidation, it is unlikely that a person will be found to be in custody within the confines
of their home absent some additional facts like handcuffing. Interrogation is found when
a question is asked that is likely to elicit an incriminating response. Here, the question
"Did you rob Jo-Jo's Bar last night?" is likely to elicit an incriminating response and is
therefore likely interrogation. However, without the custodial element, the statement of
crying could not be suppressed due to a lack of prior Miranda warnings.
2. Permitting the officer to read her handwritten notes was proper. At issue is
whether it was permissible to read these notes to the jury or if this constituted hearsay. If
a witness takes the stand and cannot remember something, they are permitted to look at
anything that may refresh their recollection provided that the opposing side has access to
the item used to refresh the witnesses' recollection. If a witness's own notes or writing is
used to refresh the witness's recollection but it is unable to successfully do so, it may be
read to the jury under the hearsay exception of past recollection recorded. A witness is
able to read their prior notes or writing if it was written contemporaneously or soon after
the event in question and if it was accurate. Here, the officer testified that he knew the
defendant said something but couldn’t remember what. He testified that the notes did not
refresh his recollection but he knew that he wrote them only two days after the incident
and that he was careful to write everything down accurately. This is enough to satisfy the
requirements for a past recollection recorded and therefore was properly admitted.
3. It was improper to admit the handwritten notes into evidence. At issue is whether
a document read as past recollection recorded may be introduced into evidence as an
exhibit. Generally, a document used to refresh a witness' recollection that is subsequently
read as past recollection recorded may only be introduced into evidence by the opposing
side. Here, the prosecution's witness testified regarding the contents of the notes and then
the prosecution asked for the notes to be received into evidence. This was improper
procedure as only the defense could introduce this into evidence should they choose to do
so.
4. It was proper to admit the testimony regarding the defendant's statement, "I have
some information that can really help you with this case." At issue is whether admission
violates defendant's Miranda rights. As discussed above, statements made in violation of
a defendant's Miranda rights should be suppressed. However, as discussed above
statements are only suppressed if they are the product of custodial interrogation. This
means that if a defendant voluntarily says something not in response to questioning or
actions by the police, that statement is not going to be suppressed under Miranda because
it was a voluntary statement not made in response to interrogation. Police must
scrupulously honor invocations of Miranda, but after voluntarily reinitiating discussions
with the police suspects may waive Miranda after having previously waived it. Here,
even though the defendant had invoked Miranda, he spontaneously stated as he arrived at
6
the police station that he had information the police wanted. He was then re-read
Miranda and at that point waived his rights and made the statement in question. The
defendant was aware of his rights and waived them voluntarily. Conversation was
reinitiated by the defendant and not the police - the police scrupulously honored his
invocation by reminding him of his rights before he continued. This knowing and
voluntary waiver was valid and therefore the statement is admissible
5. It was proper to admit the defendant's statement "I was there on November 30 and
saw the robbery, but I had nothing to do with it. After a cooling off period, police are
allowed to reinitiate interrogation and reissue Miranda warnings prior to formal charges
being brought, despite a previous invocation of Miranda. Here, the defendant invoked his
right under Miranda to a lawyer on December 2. Then over two weeks later he was
arrested and reread Miranda. The defendant knowingly waived his rights at that point
and therefore the interrogation that followed was not Miranda deficient.
ANSWER TO MEE 2
1) The decision by the trial court to admit the officer's testimony that the defendant
started crying was proper. At issue is whether the testimony violates the bar against
hearsay, or the defendant's Miranda Rights.
Hearsay is an out of court statement offered to prove the truth of the matter
asserted. The statement does not have to be verbal, non-verbal acts can also qualify by
statements if they are intended to be an assertion of fact (for example, nodding one's head
in response to a question). However, statements by a party are not hearsay when offered
against that party at trial. In this case, the defendant bursting into tears in response to the
question "Did you rob Jo-Jo's Bar last night?" is likely not a statement, and thus not
hearsay, because it was not intended to be an assertion of fact in response to the question,
but rather was a spontaneous physical act. This point is highlighted by the defendant's
continued denial of involvement in the robbery, it was not intended to be an affirmative
statement in response to the question. Thus the crying is not hearsay but is rather is a
physical observation of the officer that constitutes circumstantial evidence of the
defendant's guilty mind. Even if the crying was held to be a statement asserting the
defendant's guilt, it still would be admissible, as it is a statement by the defendant offered
against the defendant at trial and thus is not hearsay as defined by federal rule 801(d)(2).
The defendant's Miranda Rights were also not violated because the defendant was
not yet in custody. Miranda warnings need only be given to a defendant during a
custodial interrogation, and an individual is not in custody for the purposes of
interrogation when a reasonable person would feel free to leave and end questioning at
any time under the circumstances. While the officer was asking the defendant about his
7
potential involvement in the robbery, the questioning was conducted in the defendant's
home. The defendant himself invited the officer inside. The defendant would not
reasonably feel that he was not free to leave or end questioning in these circumstances, as
the defendant himself invited the officer in and could easily ask him to leave. Therefore
the defendant was not in custody at the time and the questioning was permissible without
Miranda warnings.
2) The decision by the trial court to allow the officer to read her handwritten notes to
the jury was proper. At issue is whether the officer could not remember the events of
December 1st. When a witness has trouble remembering a previously known fact, the
attorney may refresh the witness's recollection by allowing the witness to view some
document or object to jog their memory. The document itself does not need to be
admissible, only an aid to the witness's memory. After reviewing the document, if the
witness can now recall the forgotten events, the attorney would then instruct the witness
to set the document aside and to testify to the fact from her own memory. However, if
after reviewing the document, the officer still cannot recall the previously known fact, the
attorney may then read to the jury a previously recorded recollection of the events made
by the witness at a time when she could accurately recall the events in question. Here, the
witness testified that she could not remember what was said by the defendant at the
police station. Even after being shown a copy of her notes, the officer still could not
recall the events, but testified that the notes were an accurate reflection of the events
written at a time when she could recall them. As a result, the notes are a recorded
recollection, an exception to the bar against hearsay, and may be read to the jury.
3) The court erred in admitting the officer's handwritten notes as an exhibit in
evidence. At issue is when a document admissible as a recorded recollection may be
entered as an exhibit itself. While it is permissible to read a recorded recollection to the
jury under the circumstances described in question 2, entering the exhibit itself is not
allowed by the proponent of the evidence. The document may be admitted on cross
examination, but only on cross examination. Here, the document was entered by the
prosecution, the proponent of the evidence, over the objection of the defense. This makes
the exhibit improper and the judge's ruling erroneous.
4) The court was proper in admitting the defendant's statement "I have some
information that can really help you with this case." At issue is whether the statement
violates the bar against hearsay or the defendant's Miranda Rights. As previously
discussed, hearsay is an out of court statement offered to prove the truth of the matter
asserted, however statements made by a party offered against that party at trial are not
hearsay under rule 801(d)(2). Here, the statement is made by the defendant and offered
against the defendant at trial, therefore it is not hearsay.
The defendant's Miranda Rights also were not violated because the defendant
himself reinitiated questioning. After a successful invocation of one's Miranda Rights, the
8
police must cease questioning and may not reinitiate questioning to try to get a waiver out
of the defendant until a sufficient amount of time has passed. Here, the defendant did
successfully invoke Miranda at his home and the officer ceased questioning. However, at
the station, the officer did not attempt to reinitiate questioning. Instead, the defendant
himself offered information to the officer unprompted. At that point, the officer was free
to reread the Miranda Rights and obtain a waiver, which she did. The statement in
question came after this waiver, and does not violate the defendant's rights by being
admitted at trial.
5) The court was proper in admitting the defendant's statement "I was there on
November 30 and saw the robbery, but I had nothing to do with it." At issue is whether
the statement violates the bar against hearsay or the defendant's Miranda Rights. For the
same reasons discussed in question 4, the statement here is made by the defendant and
offered against the defendant at trial, therefore is a statement by a party opponent and not
hearsay.
The defendant's Miranda Rights were also not violated because a sufficient time
had elapsed between the conclusion of his last invocation of his rights to reinitiate
questioning. After a defendant invokes his Miranda Rights, officers may not attempt to
reinitiate questioning until after 14 days, as ruled by the Supreme Court. After that time
elapses, officers may again try to question the defendant, and if a new waiver of Miranda
is granted, the evidence of that conversation will not be barred as a violation of the
defendant's rights. In this case, the defendant asked for and received a lawyer on
December 1st. The police ceased questioning at that time. Questioning was reinitiated on
December 20th, more than 14 days later. At that time, the defendant waived his rights and
as a result his statement is admissible against him. It should also be noted that the
questioning does not violate the defendant's sixth amendment right to counsel, despite
being provided with a lawyer previously, as no formal charges were yet brought against
the defendant.
ANSWER TO MEE 3
1. The physician is not liable to the man under tort law. At issue is whether the
physician breached any duty to the man. In order to be liable in negligence, four things
are required: (1) the defendant must have had a duty to the plaintiff, (2) the defendant
must have breached that duty, (3) the defendant's act must have caused the plaintiff's
harm, and (4) the plaintiff must have suffered damages. A doctor owes to his patients the
duty to act as a state-of-the-art practitioner would, and is liable for breach of that duty.
Here, however, the physician's behavior did not breach that duty. His comment to the
patient that herbal tea had been seen to have a beneficial impact on cholesterol was in
keeping with a research report he had heard of, and there is no sign that he was wrong to
9
rely on that report. Moreover, his treatment of the man's inflamed liver with tests and
referral to a specialist is in keeping with standard medical practice. It does not appear
that he negligently failed to diagnose the illness's source: even liver specialists were only
able to discover the pesticide contamination as a result of seeing several patients
suffering from the same ailment. Even had the physician negligently failed to diagnose
the illness, the man would need to show that earlier diagnosis would have prevented the
liver damage in order to recover, because otherwise the physician's failure to diagnose
did not cause the injuries he complains of. Therefore, the physician is not liable to the
man under tort law.
2. None of the five U.S. companies that processed, packaged, and sold the herbal
tea to the health food store will be liable to the man under tort law. At issue is
whether a product that varies from batch to batch is susceptible to suit on a theory of
market share liability.
Market share liability is a doctrine that arises in the context of strict products
liability. Under the doctrine of strict products liability, a commercial seller of products is
liable for harm caused by a defective product that leaves its hands in an unreasonably
dangerous condition. Here, contaminated food products are unreasonably dangerous by
manufacture, in that they deviate from the expected form of a product in a way that (1)
cannot be prevented by the exercise of reasonable care, and (2) is likely to cause harm to
consumers of the food. Strict liability replaces the requirements of breach and duty from
the world of negligence, but a plaintiff suing under a theory of strict liability still must
establish causation and damages. While the man will be able to show that he was harmed
by tea, he will not be able to show causation against any one of the tea companies,
because he will not be able to show which tea company's tea caused his injury.
When there is a line of products that is uniformly dangerously defective, market
share liability is a theory that permits consumers to hold the companies that produced that
product strictly liable, even in the absence of causation. Market share liability is
appropriate only when the product in question is truly fungible: that is, that each sample
of the product is equally dangerous to each other sample. Here, that is not the case,
because the tea was contaminated in different amounts, according to how contaminated it
was, what percentage of each package sold was made up of contaminated tea, and other
factors. Therefore, the man will be able to recover in tort only if he can prove, of a
particular company, that it caused his harm. Here, that will be impossible, because he is
unable to determine which brands of the herbal tea he bought, and which of the brands of
herbal tea were contaminated. Therefore, none of the processing companies are liable to
the man under tort law.
3. The health-food store is liable to the man under tort law. At issue is whether a
consumer must identify particular products sold by a commercial seller in order to
recover. Like the processing companies, the health-food store sold contaminated herbal
10
tea. Unlike the processing companies, it is certain that the health-food store sold the
defective tea that caused the man harm. The man will have no trouble showing that the
store caused his harm, because he can prove that that store was the only source from
which he purchased herbal tea. There is no requirement that the man identify the
particular product in question; he need only show that (1) the store sold a dangerously
defective product, and (2) that product caused him harm. Therefore, the store sold a
dangerously defective product, which caused the man's harm. Therefore, the health-food
store is liable to the man under strict products liability.
ANSWER TO MEE 3
(1) Man v. Physician
The physician is probably not liable to the man under a theory of negligence. The
issue is whether it was foreseeable that the herbal tea that the doctor told the patient about
would be contaminated.
A showing of negligence requires: (1) a duty: (2) that was breached; (3) and that
the breach was the actual and proximate cause of; (4) damages. Duty imposes on every
individual a responsibility to conform to a specified standard of care in order to avoid
unreasonable risk of harm to potential plaintiffs. Here, the doctor has a duty to act as an
average professional doctor in the country would--since there is a tendency to look at the
national standard rather than at a local standard. He did conform to this duty by telling
the man that his cholesterol was high and that he should take a drug that lowers
cholesterol, but since the man insisted on trying "natural remedies" first, the physician,
after much insistence that prescription drugs are much more effective, referred the man to
a report that an herbal tea has been shown to reduce cholesterol levels in a recent research
report. He could be considered to have breached his duty by relying on alternative
medicine since he is a licensed physician. Though this argument is weak considering the
fact that alternative remedies such as teas can be recommended even by physicians.
However, three months later, the man had an increased white blood cell count as well as
an inflamed liver. At this point, it could be argued that the doctor breached a duty by
failing to tell the man to stop drinking the herbal tea. Although, generally, there is not
duty to act affirmatively, such a duty can arise when the actor is the one that caused the
peril to begin with (here, the doctor showed the man the report about the herbal tea) or
when there is a special legal relationship (doctor-patient). He had a blood test just three
months prior and did not have any of the results that he had in the second blood test. The
only change that the man made, besides a healthier diet, was drinking that herbal tea.
Therefore, it could be argued that the doctor breached his duty to conform to the national
standard of care of a physician by failing to tell the man to stop drinking the tea.
11
The man must also proof, in addition to a breach of duty, that the breach was the
actual and proximate cause of his injuries. Actual cause requires a showing that but for
the breach, the injury would not have occurred. Proximate cause requires a showing that
the injury was within the increased risk and natural incidents of the defendant's conduct.
Although but for the doctor referring him to the herbal tea and failing to prevent him
from continuing to drink the tea and thereby exacerbating his injuries, the man would not
have been injured, it is not likely that the man could demonstrate that his injuries were
foreseeable. Herbal tea is used all the time by ordinary people. The result that occurred,
liver damage, is highly unusual and therefore would probably be considered by any court
to be unforeseeable.
Therefore, a court will probably not find the doctor liable under a theory of
negligence due to a lack of proximate cause.
(2) Man v. Five US Companies
The companies are not liable to the man under any tort theory. The issue turns on
whether he can hold them liable under strict liability since they have been found to be not
negligent by investigators.
In order to find a distributor liable under a strict liability theory of products
liability, there must be: (1) a commercial seller; (2) a defective product; (3) that was
defective when it left the hands of the defendant; (4) that was foreseeably used; (4) that
caused; (5) damages. Here, the five companies are commercial sellers of the herbal tea.
The product would be considered to be a manufacturing defect since some of the teas had
contaminants and some did not. The product was defective when it left the companies'
hands since research shows that the contamination occurred in the export warehouses of
Country X, already by the time it reached the US distributors. The product was used
foreseeable, to drink. The product actually caused the man's liver damage since this is
undisputed. However, the man will be unable to prove which of the US distributors were
responsible for the herbal tea that he drank. Therefore, though he actually suffered
damages, he will be unable to hold them liable unless he could prove which of the US
companies supplied him his herbal tea. However under one theory of causation from
Summers v. Tyce, when all defendants are culpable, the burden would shift to them to
show that each was not culpable for that particular product. However, that theory was
used in a negligence case, and here, it's a strict liability theory. Moreover, the
investigators concluded that the companies were not negligent. In addition, some
packages had no contaminants whatsoever. Therefore, given the inability to pinpoint
which of the companies supplied the man his tea, he will unlikely be able to find them
liable as a group.
12
(3) Man v. Health-Food Store
The health food store is likely liable under a theory of strict liability.
The elements for a strict liability for a product defect are the same as above. The
store is a commercial seller that sold a defective product that was defective when it left its
hands. The tea was foreseeably used since it was consumed by drinking and the tea
caused the man's liver damage. Here, although the man is not sure which brand he
purchased, he knows that the tea he purchased from was from this one health food store.
Therefore, there is no causation issue as there was for the five US companies.
The health food store is therefore liable under a theory of strict products liability to
the man. As a side note, it should be noted that the health-food store not likely be found
liable under a theory of negligence. Those further down the chain of distribution can
escape negligence liability usually through cursory inspection of the item. A cursory
inspection of the herbal tea would not reveal the contamination.
ANSWER TO MEE 4
1)
The issue is whether disabling PTT's proton-therapy equipment would be a viable
remedy for Ion to pursue.
When a debtor defaults on a security agreement, a secured creditor may pursue a
variety of remedies. The creditor may initiate a replevin action to acquire the property, or
it may use self-help to repossess the property provided that this does not lead to a breach
of the peace. In many jurisdictions, the debtor's mere presence and objection to a self-
help repossession is enough to compel the creditor to use legal process. In line with the
general disfavoring of self-help methods, a creditor may not extra-legally destroy or
deactivate its collateral, particularly when such actions are likely to lead to large
consequential damages for the debtor, and may forfeit important rights in any subsequent
sale of the property.
In this case, Ion would propose to surreptitiously disable (what could be life-
saving) cancer equipment under the guise of performing a routine maintenance. This
action is not reasonably calculated to allowing Ion to recoup its security interest; on the
contrary, it is likely to cause severe and unforeseen consequential damages to PTT and
may even be tortious. As such, Ion would quite possibly be held liable to PTT for such
actions, and should use legal process to recover its security interest in the collateral.
13
2) The issue is whether Bank or Ion has a superior claim to the proton- therapy
equipment.
a) Ion's Interest in the Proton-Therapy machines
In order for a security interest to attach, a debtor must (1) have paid value; (2)
have rights in the collateral; and (3) there must be a security agreement describing the
collateral or the creditor must have possession or control over the collateral. A valid
security agreement must be a record authenticated by the debtor that "reasonably
describes" the collateral. A transaction in which a seller transfers property to a buyer and
retains a security interest in that property as collateral is called a seller "purchase money
security interest" (seller PMSI). Tangible property in machinery or other products used
for the ordinary functioning of a business is called "equipment". A PMSI in equipment
can be perfected by filing a financing statement which gives describes the collateral and
gives the name of the debtor in the appropriate state office. PMSIs in equipment will take
priority over all other security interests if a financing statement is filed within 20 days of
the debtor taking possession of the property.
In this case, Ion sold proton therapy equipment to PTT and retained a security
interest in the equipment as collateral, thus creating a seller PMSI in equipment. Its
interest attached upon the execution of the security agreement, and perfected when it filed
financing statement with the state office, giving the proper name of the debtor and
adequately describing the collateral.
b) Ion's Interest in the Equipment After it Became a Fixture v. the Bank's Interest
in the Facility
After property becomes a fixture, an interest in the property exists in favor of
anyone with an interest in the real property. A holder of a PMSI in the fixture may retain
priority over a mortgage holder by a filing a fixture financing statement in the proper real
estate records office within 20 days of the property being incorporated as a fixture. If it
fails to do so, it retains its security interest, but it is subordinated to the interest of the
property's mortgage holder.
In this case, though Ion had a PMSI in the equipment, it knew that the equipment
was going to be installed as a fixture in the facility and yet failed to file a fixture
financing statement in the real estate records office. Therefore, the Bank has priority over
Ion in the equipment, which is now a part of the greater facility. Of course, the Bank's
interest in the equipment only goes to the extent of the facility's outstanding obligation,
and any surplus would still go to Ion.
14
3) The issue is whether Ion has an enforced and perfected security interest in any
other PTT assets.
A secured creditor's interest automatically attaches in the proceeds of the
collateral. While an interest in cash proceeds also perfects automatically, perfection can
be maintained in non-cash proceeds via the "same office rule" - if a financing statement
for the proceeds can be filed in the same office where the financing statement for the
initial collateral was filed, then perfection is maintained throughout. Proceeds can be the
product of a sale or a lease of the collateral, and can be either tangible or intangible, the
latter including accounts and rights to payment.
In this case, PTT contracted with Oncology for its use of the equipment for a
period of time, and on the latter's default acquired an interest in a right to payment from
Oncology. This right to payment could be characterized as a "proceed" of the equipment
in which Ion had a PMSI. Therefore, provided that an interest in PTT's right to receive
payment from Oncology could be perfected by filing in the same office as the equipment,
Ion would have an enforceable and perfected security interest in any recovery that PTT
may receive from Oncology on the lease.
ANSWER TO MEE 4
1. Ion can disable the proton-therapy equipment.
When a debtor defaults on his payments under an attached security agreement, the
secured creditor can repossess the collateral using self-help methods as long as he does
not breach the peace. For equipment that is too bulky to be removed for repossession, the
secured creditor may render the collateral unusable. A security agreement attaches when
(1) the secured party gives value; (2) a security agreement is authenticated by the debtor;
and (3) the debtor has rights in the collateral. Here, Ion (the secured creditor) provided
PTT (the debtor) with the proton-therapy equipment, PTT signed a purchase agreement
that provided for a security interest, and shortly thereafter Ion delivered the equipment.
Therefore, the security interest attached when the equipment was delivered. It is
undisputed that PTT is in default. Therefore, as long as Ion does not breach the peace, it
can render the equipment unusable in order to protect its security interest.
2. Bank has a superior claim to the proton-therapy equipment.
The issue is whether Bank's purchase money mortgage has a superior claim to
Ion's PMSI in equipment that is a fixture. Generally, security interests in fixtures are
considered junior to a security interest in real property unless the security interest in the
fixture was a purchase-money security interest (PMSI) and that interest was perfected
15
within 20 days of installation. In order to perfect a security interest in a fixture, the
interest must be recorded in the county real estate records office. Generally, PMSIs in
equipment have priority over other security interests so long as the security interest was
perfected before delivery AND the PMSI secured creditor sent notice to the other secured
creditors. Here, state law states the equipment is a fixture and in favor of anyone with an
interest in the building.
Perfection is required to put other secured creditors on notice that there is a
security interest. Without perfection, a secured creditor cannot assert .Bank's mortgage
interest is properly recorded and therefore perfected. Ion did not file a statement for the
fixture in the county real estate records office, so therefore it is not perfected against
Bank's mortgage interest. Ion also did not send notice to Bank of its security interest.
Therefore, Bank's interest in the equipment is superior.
3. Ion has an enforceable and perfected security interest in Oncology's lease
payments.
The issue is whether the amount due on the Oncology lease can be considered
proceeds of Ion's equipment. Generally, a security interest automatically attaches to
proceeds. This interest must be perfected. Under the same-office rule, if a security
interest in the type of proceed can be perfected by filing a statement in the same office as
the original collateral, that proceed is automatically perfected. The right to receive
payments for a lease is considered an account. An account can be perfected by filing a
statement in the statewide filing system. Since Ion has filed a statement for the equipment
in the statewide filing system, its interest in the payments due on the Oncology lease is
perfected.
ANSWER TO MEE 5
1. Neighbor v. Painter
The neighbor would succeed in a breach of contract action against the painter. At
issue is whether contracts are assignable absent any provision to the contrary. In general,
contracts are assignable unless the contract itself provides otherwise, or the assignment or
delegation creates a material change in the duties of the promisor. Once the contract has
been assigned, the assignee may sue either the assignor or the promisor upon a breach.
Here, the contract does not provide that assignments are not permitted, or are void.
Therefore, the homeowner, who has promised to pay $6,000 in exchange for having his
house painted, can delegate his rights and duties under the contract to his neighbor, as
long as there is no material change in the duties of the painter. There is no material
change in the duties of the painter because the houses' exteriors are identical, have the
16
same level of wear, and there is no difference in magnitude or difficulty between the
work required to paint the two houses. Therefore, because there is no material change in
duties, and the contract does not provide otherwise, the neighbor will succeed in a breach
of contract action against the painter.
2. Retiree v. Painter
The retiree would not succeed in a breach of contract action against the painter. At
issue is whether he is an intended or incidental third party beneficiary. Third party
beneficiaries may have causes of action for breach of contract, but only if they are
intended beneficiaries, and their rights under the contract vest. An intended beneficiary is
one specifically for whose benefit the contract is made and, generally, an intended
beneficiary is named in the contract. Here, the contract was not made for the retiree's
benefit - it was made first for the homeowner's benefit, and then for the neighbor's.
Furthermore, the retiree is not named in the contract. Therefore, the retiree is an
incidental beneficiary and will not have a cause of action against the painter.
3. Painter v. Neighbor and Homeowner
The painter would succeed against both the homeowner and the neighbor. At issue
is whether a novation has occurred. When a contract has been assigned, generally the
assignor remains liable on the contract until there has been a novation - or an agreement
between the assignor and the promisor to entirely replace the assignor with the assignee,
essentially creating a new contract. However, the assignee also becomes liable on the
contract when he accepts the assignment. Here, there has been no novation - in fact, the
painter has specifically objected to the substitution of the neighbor for the homeowner in
the contract. However, the neighbor has accepted the assignment, so he is also liable.
Because there has been no novation, both the homeowner and the neighbor could be
liable to the painter.
ANSWER TO MEE 5
Painter Refuses:
The neighbor would likely not succeed against the painter, though it is a close call.
At issue is what duties may rightfully be delegated.
A contract is formed by mutual assent, exchange of legally bargained for
detriment or benefit, i.e. consideration, and promised performance. Here, the painter and
the homeowner had a valid contract because they both assented to the terms, the price,
17
the conditions, and the schedule. This is a contract for services and so the common law
governs.
A party to contract is generally free to assign either the rights of the contract or to
delegate the duties, or both, without further consideration or consent from the other party.
Assignees can enforce a contract, especially if they have paid consideration for it. But
rights and duties must not be delegated if they would substantially change the nature of
the contract. Courts will usually hold the nature of the contract to not be substantially
changed if the person receiving payment has changed, but usually will find substantial
change if the performance has been altered, even if substantially similar. Here, the
principle is tested: if a court is harsh and views the above rule strictly, the right to
have a house painted will not be able to change to a new, if almost identical house
(nearby location, similar appearance, similar magnitude and difficulty); if a court is more
lenient, it will allow the neighbor to transfer the right to an extremely similar house, as
the painter's duties would not seem to differ much or at all from the change. So while the
neighbor-assignee would have a right to succeed in a breach action generally, the
outcome here will differ based on how stringently a court enforced the substantial change
rule for the nature of performance.
Alternatively, if it is deemed a modification, it is also invalid. Common law
modifications require new consideration and agreement between the original parties.
Here, there is consideration between the new party and one of the original parties only.
And there is no agreement with the painter. Thus, the change is invalid as a modification.
Some courts hold now that common law modifications are allowed if the deal is
still fair and equitable in light of unanticipated circumstances. Notwithstanding the above
problem, there appear to be no truly unanticipated circumstances. The law contemplates a
natural disaster, not a conversation with a neighbor, as the type of event in mind to trigger
this trend.
Based on the discussion just below, the neighbor was also never a third party
beneficiary and thus does not have that avenue of enforceability either.
Retiree:
The retiree will not succeed in a breach action. At issue is what constitutes a third
party beneficiary.
Third party beneficiaries are those who are intended by the parties to benefit at the
outset of the contract. They can then enforce the contract once they assent to the contract,
detrimentally rely on it, or seek to enforce their rights under it. Here, the retiree, while he
has a general neighborhood interest in the paint job, was not explicitly mentioned in the
contract or negotiations with the painter as an intended beneficiary. Though he does stand
18
to benefit from better property values, that general benefit is insufficient for official 3P
status. Nor was the retiree specifically in the mind of the neighbor when he paid the
consideration for his house to be painted. The neighbor was thinking mainly of his
relatives and only remembered the retiree later, after the contract was made. That the
retiree was likely part of the reason for the motivation is similarly insufficient for official
status--he must have been mentioned during the contract or at least negotiations, and
here, he was not.
His statement of "about time" may have been minimally sufficient for assent and
thus enforcement had he been an intended beneficiary, but it is a moot point because he
was not specifically designed to be benefited by either the neighbor's nor homeowner's
contract and thus cannot succeed in a breach action.
Painter Paints:
The painter could succeed against either party. At issue is who a wronged obligor
can sue.
When assignees are given full rights and duties under a contract, they must fulfill
their duties to receive their rights. Being assigned "the contract" means taking both rights
and duties. Here, the assignee accepted "the contract", and for consideration, and thus
took both rights and duties. If he does not fulfill the duty of payment, the painter must sue
him to enforce his right to be paid against the neighbor.
Assignors generally remain liable for contracts that they assign. Here, the home
owner assigned the contract but remains liable.
Novations remove liability from an original party. Novations require agreement of
both parties, the end of the old contract and the formation of a new one. Assignment is
not novation. Here, the parties talked about the neighbor taking the contract, meaning the
existing contract. There was no talk of a new contract, and anyway the painter never
agreed. Thus, there was no novation that would remove original party liability, only
assignment. Therefore, the home owner remains liable in addition to the assignee
neighbor.
ANSWER TO MEE 6
The State C court has personal jurisdiction over the man. The issue here is whether
serving someone with a summons and complaint in the state in which personal
jurisdiction is sought is sufficient to confer personal jurisdiction over that person. States
may exercise personal jurisdiction over defendants if they are authorized to do so by
19
statute and such exercise comports with the constitutional requirements for personal
jurisdiction. The Constitution requires that the defendant have sufficient contacts with the
state so that exercise of personal jurisdiction over him is fair and reasonable. Specific
jurisdiction involves a connection between the defendant and the particular claim at issue.
General jurisdiction requires that the defendant be at home in the state at issue, meaning
that he is domiciled there. However, personal jurisdiction is always proper in a state in
which the defendant voluntarily appears and is served with a complaint and summons.
There is an exception for visits to the state which are done to testify at a court proceeding.
Here, the man deliberately visited State C, where he was served with a complaint
and summons. The District Court in State C is trying to exercise personal jurisdiction
over him. State C has a long-arm statute, so we consider only the constitutional aspect of
the personal jurisdiction inquiry. State C does not have general jurisdiction over the man,
since his domicile is State A, which is the only state with general jurisdiction over him.
Turning to specific personal jurisdiction, although the man's visiting state C is unrelated
to the woman's claim (after all, the events giving rise to the claim occurred in State A; he
was merely visiting her at the hospital to which she was airlifted in State C), personal
jurisdiction is clearly satisfied here because the man voluntarily visited State C and was
served with a complaint and summons there. When a defendant voluntarily visits a state
and is served in that manner, the courts of that state automatically have personal
jurisdiction over him. Therefore, the court in State C may exercise personal jurisdiction
over the man.
Next, the State C court does have subject-matter jurisdiction over the woman's
case. The issue here is whether the woman's state-law claim can be brought into the case
under supplemental jurisdiction given that the court already has jurisdiction over the
woman's federal law claim. Federal courts have limited subject-matter jurisdiction. There
must be either diversity jurisdiction, meaning that no plaintiff is from the same state as
any defendant, and the amount in controversy exceeds $75,000, or federal question
jurisdiction, meaning that the plaintiff's well-pleaded complaint states a claim arising
under federal law. However, if a case contains a claim under federal question jurisdiction
and another claim that fails to meet diversity jurisdiction, the latter claim may be brought
into the case under supplemental jurisdiction. Supplemental jurisdiction exists where the
claim, which fails diversity or federal question jurisdiction, arises from the same nucleus
of operative fact as the claim that is properly subject to federal-court jurisdiction. A court
may refuse to entertain the state-law claim if it deems it inappropriate to do so, because
the state-law claim is very complex or for other reasons. The only exceptions to
supplemental jurisdiction apply in cases of diversity jurisdiction, which is not applicable
here.
Here, the State C court clearly has federal question jurisdiction over the woman's
federal claim under the Safety Act. That claim involves the woman attempting to assert a
federal right against the man for harming her as a result of an alteration of the safety
20
features of a gun that has traveled in interstate commerce. The claim therefore arises
under federal law, and it can be brought in the State C District Court. However, the man's
state-law negligence claim is neither a federal question claim nor a diversity claim. It
asserts no federal rights, and it fails to satisfy diversity because, while the amount in
controversy is $120,000, both the man and the woman are citizens of state A. There is
thus no diversity here, and so the claim can come in only as a matter of supplemental
jurisdiction. Crucially, the state-law claim arises from the same transaction or occurrence
as the federal claim which is already able to get into federal court, because the negligence
claim is based on the same event, namely the man's alteration of the safety feature and the
ensuing injury during the hunt. Moreover, there is no reason to not try the two claims
together, since this seems like a standard negligence claim. Moreover, the exceptions to
supplemental jurisdiction are inapplicable, because the other claim is a federal question
claim, not a diversity claim. Therefore, the State C court as subject-matter jurisdiction
over the woman's entire case.
Finally, venue is not proper in State C District Court. The issue here is whether
venue is proper in a state where the victim of an accident receives treatment for injuries
suffered in another state. Venue is proper in those districts where a substantial part of the
events giving rise to a claim occurred, or in any district where all defendants reside. Here,
the man resides in State A, so venue in State C would not be proper on that basis. The
only way in which the State C court could be the proper venue would be if a substantial
part of the events giving rise to the claim occurred in State C. However, that is simply not
the case. After all, the event giving rise to woman's claim was the injury she suffered
from the man's alleged negligence. That happened in State A. It is the injury itself that
creates liability, not the treatment. The claim would've accrued even if the woman had
been treated in some other state. Therefore, since a substantial part of the events giving
rise to the claim did not occur in State C (they only occurred in State A, or possibly State
B), State C cannot be the proper venue for this action.
ANSWER TO MEE 6
Lack of Personal Jurisdiction
The State C court can properly assert personal jurisdiction over the man. The issue
is whether service of process in State C, without other sufficient contacts, is enough for
the court to exert personal jurisdiction over the defendant.
Personal jurisdiction describes a court's power over parties to a litigation. For
personal jurisdiction to be proper, a court's exercise of jurisdiction must both:
1) Be proper under the state's long-arm statute, and 2) Comport with the constitutional
requirements of the Due Process Clause, which only allows courts to assert jurisdiction
21
over a defendant if the defendant has such minimum contacts with the state such that the
exercise of jurisdiction would not offend traditional notions of fair play and substantial
justice. Under the due process clause, if a defendant is "at home" in the forum state,
usually meaning the forum state is the state of domicile if the defendant is an individual
or the state of incorporation or principal place of business if the defendant is a
corporation, the court may assert general jurisdiction, i.e. jurisdiction over claims against
the defendant arising from anywhere. If a defendant is not at home in the forum state, but
his contacts with the state demonstrate purposeful availment of the privileges and benefits
of the state's laws such that the defendant could reasonably foresee being hauled into
court in the forum state, the court may assert specific jurisdiction, i.e. jurisdiction over
claims arising from the defendant's activity in the forum state (the exercise of specific
jurisdiction must also be "fair," examining factors such as whether the exercise of
jurisdiction would be so grossly inconvenient for the defendant as to deny him due
process, the forum state's interest in the litigation, the interstate judicial system's interest
in the efficient resolution of controversies, and the plaintiff's interest in litigating in a
convenient forum). Ordinarily, service of process on the defendant within the state is, in
itself, sufficient to confer personal jurisdiction, even if the above tests are not met, though
states usually grant immunity from this rule for defendants who are in a state solely to
testify in another judicial proceeding.
Here, the state's long-arm statute specifies that a court may assert personal
jurisdiction so long as the exercise of jurisdiction is allowed under the U.S. Constitution,
so the only question is whether the court's exercise of jurisdiction complies with Due
Process. The defendant's contacts with State C would clearly not be sufficient to confer
jurisdiction under the ordinary minimum contacts analysis. He is not "at home" in State
C, as he has never visited there until the visit in which he was served process, so general
jurisdiction would not be appropriate. Furthermore, specific jurisdiction would be
inappropriate, as his only contact in State C is the trip in which he went to State C to
conduct business and visited the woman in the hospital, and none of the events giving rise
to the litigation arose out of that visit (in fact, they happened well before). However,
because the man was served process by the woman's attorney while visiting her in the
hospital in State C, the court may properly exercise personal jurisdiction. Note that, even
if this state grants immunity to defendants who are in the state solely for judicial
proceedings, this immunity does not apply here because the defendant was in State C for
business, not to testify in a proceeding.
Therefore, the exercise of personal jurisdiction is proper.
Subject-Matter Jurisdiction
A court can properly exercise subject-matter jurisdiction over the claims at issue.
The issue is whether the requirements for federal question, diversity, or supplemental
jurisdiction are met for both claims.
22
Federal courts are courts of limited jurisdiction, meaning they generally may only
hear cases for which they have been granted subject-matter jurisdiction by statute. By
statute, federal courts have been granted jurisdiction in two main types of cases: 1)
Federal question cases, and 2) Diversity cases. In addition, a court may exercise
supplemental jurisdiction over claims arising from the same core nucleus of operative
facts as a case over which it has subject matter jurisdiction.
Federal Question
A court may exercise federal question jurisdiction if the plaintiff's claim arises
under the Constitution, laws, or treaties of the United States. Under the well- pleaded
complaint rule, a federal cause of action must be clear on the face of the plaintiff's
complaint, i.e. the plaintiff must be asserting a federal right. A federal defense, or an
argument based on federal law in anticipation of a defense in plaintiff's complaint, is not
sufficient to confer jurisdiction.
Here, the plaintiff's federal claim under the Federal Gun Safety Act clearly arises
under a federal statute, while her state law claim for negligence clearly arises only under
state law. Therefore, the court has federal question jurisdiction over the FGSA claim, but
not the state-law negligence claim.
Diversity
Diversity jurisdiction is proper when: 1) There is complete diversity, i.e. each
plaintiff is from a different state from each defendant, and 2) The amount in controversy
exceeds $75,000, exclusive of interests and costs. A plaintiff may aggregate her claims
against a single defendant to meet the amount in controversy requirement, and a case will
only be dismissed on amount in controversy grounds when it is clear to a legal certainty
that the plaintiff cannot meet the requirement.
Here, the amount in controversy requirement is clearly met, as the plaintiff seeks
$220,000 in damages when both causes of action are combined (in fact, each action
individually would be sufficient to meet this requirement), and there is no suggestion that
these sets of damages are not pled in good faith. However, complete diversity is lacking.
Individuals' "citizenship" for diversity purposes is determined by the individuals'
domicile, and here, both the plaintiff and the defendant are domiciled in State A.
Therefore, the court cannot properly exert diversity jurisdiction here.
Supplemental Jurisdiction
As stated above, a court may exercise supplemental jurisdiction over claims
arising from the same core nucleus of operative facts as a case over which it has subject
23
matter jurisdiction. A claim arises from the "same core nucleus of operative facts" when
it arises from the same transaction or occurrence as a claim over which the court has
jurisdiction.
Here, the state-law negligence claim arises from the exact same incident as the
Federal Gun Safety Act claims, and the FCSA does not preempt the state law cause of
action. Additionally, while there are limits on supplemental jurisdiction when a plaintiff
whose claim is in federal court based solely on diversity attempts to defeat the
requirements of complete diversity by using supplemental jurisdiction to assert a claim
against impleaded or mandatorily joined third parties who are not diverse, these limits are
not applicable here, as the case is in federal court due to federal question jurisdiction, and
the court can thus assert supplemental jurisdiction over the plaintiff's state-law claims
against the defendant that are sufficiently related to the federal claim.
Conclusion
Therefore, the court has federal question jurisdiction over the federal law claim,
and supplemental jurisdiction over the state law claim.
Venue
Venue is improper here, and the court should transfer the case to a proper venue.
The issue is whether the court's exercise of jurisdiction meets the requirements of the
federal venue statute.
Under the federal venue statute, venue is proper in the judicial district where:
1) Any defendant resides (i.e. is domiciled if an individual or subject to personal
jurisdiction if a corporation), if all defendants are from the same state, 2) A substantial
part of the acts or omissions giving rise to the claim occurred, or 3) If no district exists
that meets one of the above requirements, where any defendant is subject to personal
jurisdiction. If venue is improper, a court may dismiss for lack of venue, or transfer to a
court where venue is proper in the interests of justice (usually, transfer is preferable).
Here, venue would be proper either in the district where the defendant resides in
State A, the district where the accident giving rise to the claim occurred in State A, or,
arguably, the district where the defendant bought the gun in State B (though this is
debatable, as the claim really arises from the accident and the defendant's alteration of the
safety features, both of which occurred in State A). State C does not meet either of the
tests laid out above; it is merely the state where the woman went to the hospital after
being injured in State A. This activity alone did not give rise to the cause of action at
issue in the case, and no defendant resides there, so it is improper.
24
Accordingly, venue is improper in the District of State C; however, rather than
dismiss the case, the court should most likely transfer the case to a district in State A
where it could have been brought, so that the plaintiff can continue to pursue her claim in
the proper forum with a legitimate interest in the litigation.
ANSWER TO MPT 1
Dear Ms. Gregson,
I the forthcoming memo I have analyzed the options available to our client, Ms.
Barbara Whirley, with regards to the issues she is having with her landlord. I have
analyzed each instance for which she was seeking repair separately.
A. Leaking Toilet
Ms. Whirley has complained that the toilet in her second bathroom of the house
she is renting has been leaking. The leak at first revealed itself with some water on the
bathroom floor, but over time the leak continued to get worse. She then had to place a
bucket by the toilet to catch the leaking water and after around two months of no repair,
the leak got to the point where Ms. Whirley had to empty the plastic bucket twice a day
and sometimes the toilet never flushed. Ms. Whirley first contacted her landlord, Mr.
Sean Spears about the leak on February 19, 2016, and continued to notify him about the
leak for the next three months. After, her landlord did nothing about it she finally called a
handyman to fix it, for which she paid $200.
Under Franklin Civil Code §541, a dwelling is deemed untenantable if it lacks (2)
“plumbing or as facilities…maintained in good working order”. A leaky toilet would thus
fall under this provision and render a dwelling untenantable. Under §540 of the Code, a
landlord of a building intended for residential purposes must maintain the building in a
way that it is fit for human occupation and must repair all subsequent conditions that
render the building untenantable. Similarly, in Burk v. Harris (Fr. 2002), the court found
that a dwelling was untenantable if a substantial breach of the warranty of tenantability
has occurred. The court cited §541 of the Franklin Code in its determination of what is
considered a substantial breach. Therefore, it is unquestionable, from both the statutory
law and the case law that the landlord breached his warranty of tenantability to Ms.
Whirley and thus was required to make the necessary repairs resulting from the breach.
With regards to the remedies available to Ms. Whirley, Franklin Code §542 has presented
four possible options available to tenants, whose landlords have neglected to repair
conditions, warranting the premises untenantable. With regards to the repairs to the toilet,
the first option is most appropriate. Option one states that if the cost of such repairs does
not exceed one “month’s rent of the premises, make repairs and deduct the cost of repairs
25
from the rent when due”. Here, the cost of fixing the toilet was #200 and her monthly rent
was $1,200. Therefore, one option for Ms. Whirley is for paying only $1,000 on her next
month’s rent, deducting the $200 cost. Furthermore, Ms. Whirley could vacate the
premises and be discharged from paying further payment, but she expressly stated that
she does not want to leave her house, so that option would not suit her. She could
withhold the rent entirely on subsection four if the conditions resulting from the lack of
repairs substantially threaten the tenant’s health and safety. It is unlikely that a leaky
toilet, although untenantable, will not reach the level of threatening Ms. Whirley’s health
and safety. Therefore, option one as noted, would be best for her. It should be noted that
the remedies available to tenants under §542 are only available if the tenant has given the
landlord reasonable amount of time to make the repairs after receiving notice of the
condition. This pre-requisite Ms. Whirley has clearly met as three months would be
considered more than reasonable time to repair a leaky toilet.
Additionally, in order for a tenant to recover for conditions in her dwelling, the
tenant must also prove that she was not at fault in causing the defective condition and
must provide documentation proving that the expenses were in fact covered by the tenant.
In Shea v. Willowbrook (Fr. 2012), the court denied a tenant’s claim for reimbursement of
rent payments when his apartment was infested with bed bugs and the tenant vacated the
premises as a result of those bed bugs. The court denied an award to the tenant because
he failed to demonstrate that the bed bug infestation was solely the fault of the landlord
and resulted from no fault of his own. The tenant in that case also failed to give the
landlord proper notice of the issue for the landlord to have an opportunity to resolve the
problem. Therefore, the court denied him reimbursement for rent payments, after he
unjustifiably vacated the premises. Furthermore, the court in that case only awarded
damages if the tenant provides proper documentation of his expenses resulting from the
unseemly condition. Therefore, he only provided $400 worth of receipts. The court only
awarded the tenant $400 for his expenses. Here, Ms. Whirley did give the landlord notice
of the need for repairs, with reasonable time for the landlord to repair and she provided
an invoice of her $200 expense for fixing the toilet. Furthermore, although no direct proof
exists that she did not contribute to the leakiness of the toilet condition, it is likely that no
reasonable tenant, which Ms. Whirley has not proven that she isn’t, would commit acts
that break their own toilet. Therefore, in summation Ms. Whirley’s best option is to
recover the $200 on her next month’s rent.
B. Outdoor Sprinkler System
Next, Ms. Whirley has requested payment for her outdoor sprinkling system that
she has paid $300 to repair. Ms. Shirley first notified her landlord about the condition on
March 31 and complained that because it does not work. She now must water her flowers
by hand 2-3 times a week, taking up about 15-20 minutes of her time. By the end of May,
the landlord had still not repaired the condition, two months later. Although she gave the
landlord a reasonable amount time to repair the condition after giving him notice, Ms.
26
Whirley will unlikely be able to recover the $300 cost in repairing the condition. A faulty
sprinkling system is not included under any of the conditions put forth in §541 of the
Code and cannot be interpreted as a substantial breach of the implied warranty of
tenantability as discussed in Burk because the condition does not make the house itself
unfit for human occupation nor does it pose any substantial risk to Ms. Whirley’s health
or safety. Although it may be annoying and now require extra time on her part to repair, it
is insufficient to recover any amount under either the relevant statutory or case law.
C. Sliding Glass Door And New Carpet
Ms. Whirley next complained that the sliding door in one of her bedrooms does
not close properly and has left a gap between the bottom of the door and the door frame,
which allows water to get into the house and dampen the carpet when it rains. Due to the
carpet’s wet conditions, mold has started to grow and a smell has resulted that is so bad
that she can no longer make any use of that bedroom. Ms. Whirley has tried to close the
door but it has not budged and has placed plastic along the door frame to try to keep the
moisture out but to no avail. This activity would satisfy her duty to try to mitigate
damages as is necessary under a tenant’s duties set forth by the court in Shea. Ms.
Whirley first notified her landlord of this condition on May 26, 2016. She then paid to fix
the condition less than a month later on June 23
rd
. Under §541 of the Code subsection (1),
this condition would likely deem her dwelling untenantable as it lack “effective
waterproofing and weather protection of roof and exterior walls, including unbroken
windows and doors”. Additionally, the condition has caused mold to grow which is not
safe and healthy, satisfying the requirements of a substantial breach as set forth in Burk.
The issue though is less than one month a reasonable amount of time for the landlord to
repair the condition. Given the danger to her health and safety resulting from this
condition, the landlord should still be responsible to pay. Under §542(c) of the Code, if
the tenant has acted more than 30 days after giving the landlord notice, she has acted after
a reasonable time and the landlord would be liable for the damages. However here, the
tenant made repairs less than 30 days after giving notice. The Code states that a tenant
may make repairs after shorter notice if the circumstances require shorter notice. Having
mold growing in your house would satisfy the circumstances requiring shorter notice
because it affects Ms. Whirley’s health and safety every day it remains. Therefore, her
landlord can still be held accountable to pay for this condition. The repair cost her $1,800
which exceeds her monthly rent of $1,200. Under §542(2), if the cost of repairs exceeds
one month’s rent, the tenant can make repairs and sue the landlord for cost of repairs, or
under subsection (4) the tenant may withhold all the rent until the landlord makes the
repairs if the conditions substantially threaten the tenant’s health and safety, which the
condition here does. However, Ms. Whirley already repaired the condition herself.
Option 2 would best suit her and thus she should sue the landlord for the $1,800.
D. Damage To Wall And Baseboard In Laundry Room
27
Finally, Ms. Whirley alleges that her dog has chewed away a two foot strip of the
baseboard in her laundry room and damaged the wall in the process and has asked if the
landlord can recover the repairs to those as well. Ms. Whirley will likely not be able to
recover for these damages. First of all, she never notified the landlord about the need to
make these repairs. Second, because it’s her dog, Ms. Whirley has a duty to make sure
her own dog does not damage any parts to her house. Her failure to do so would be partly
her fault which bars recovery under the court in Shea. She also violated Fr. Code §543 by
allowing her dog to destroy or damage part of the dwelling unit and its equipment, which
a landlord has no duty to repair. The fact that her landlord allowed to keep a dog in the
house is no defense. Therefore, Ms. Whirley will not be able to recover the $300 for
repairs that her dog caused the damage.
ANSWER TO MPT 1
MEMORANDUM
To: Della Gregson
From: Examinee
Date: July 26, 2016
Re: Barbara Whirley matter
As you noted in your earlier memo, our client, Barbara Whirley ("Whirley"),
would like to know what her options are in seeking to have several issues with her rental
house repaired. I have reviewed the relevant Franklin law, and have commented on her
various options below.
Toilet Leak
Whirley likely has several options with regards to the toilet leak; likely her best
option is to contract with JB Handyman Services to repair the toilet, and deduct the $200
cost from her rent. The Franklin Supreme Court has held that an implied warranty of
tenant ability exists in each residential lease. Gordon v. Centralia Properties Inc. (Fr.
Sup. Ct. 1975). This warranty has been codified, and Lessors such as Whirley’s Landlord
Sean Spears ("Spears") are required to put the rental building "into a condition fit for
such occupation and repair all subsequent conditions that render it untenantable."
Franklin Civil Code §540. One way in which a dwelling is deemed untenantable is if the
plumbing is not maintained in good working order. Franklin Civil Code §541(2). A
landlord should repair any such conditions within a reasonable amount of time, or the
tenant may seek a variety of remedies. Here, Whirley can show that Spears failed to
maintain the plumbing in good working order. Barbara's toilet began leaking in early
February. She originally put a towel down on the area and made affirmative efforts to
28
keep it dry. The leak got worse, and she put a bucket behind the toilet to catch dripping
water sometime in early March. By March 31, she had to empty the plastic bucket twice
a day and the toilet sometimes doesn't flush. Spears might argue that a mere leak in the
toilet is not indicative of a plumbing issue, but once the toilet stopped consistently
flushing, Whirley has a stronger argument that her plumbing was not in good working
order. Additionally, Whirley can show that Spears did not repair the plumbing condition
within a reasonable time after receiving Whirley's written notice as required under
Franklin Civil Code §542. Whirley first notified Spears, in a written e-mail, of the
leaking toilet on February 19. She notified him that the toilet had stopped flushing on
March 31. As of July 25, over 5 months after receiving written e-mail notice of the leak,
and almost 3 months after receiving written e-mail notice of the flushing issue, Spears
had still not repaired the plumbing condition.
Under Franklin Civil Code §542, because the estimated cost of the repairs ($200)
does not exceed her monthly rent of the premises ($1200), she may make the repairs
herself and deduct the $200 from her subsequent rental payment. Franklin Civil Code
§542(a)(1). Because she has given notice more than 30 days ago, she is presumed to
have waited an appropriate amount of time for Spears to repair the condition himself.
Franklin Civil Code §542(c). She doesn't wish to vacate the premises (Franklin Civil
Code §542(a)(3)) and it is unlikely that she will be able to withhold a portion of all of the
rent until Spears repairs the issue, under Franklin Civil Code §542(a)(4), because in order
to withhold a portion of the rent under this provision, the impaired condition must
threaten the tenant's health and safety. It's unlikely that the conditions of the toilet leak
alone "substantially threaten the tenant's health and safety." However, the court in Burk
v. Harris found that a shower leak, when combined with the fact that the thermostat didn't
work, and that the roof and windows leaked, together were conditions that affected
Tenant's health and safety. Burk v. Harris (Fr. Ct. of App. 2002). Thus, when combined
with the insulation problems with the sliding door, the leaking toilet could be an
additional factor that could allow Whirley to withhold the rent for all of the conditions
combined, until Spears makes all of the necessary repairs. Because he hasn't made any
repairs yet, despite her repeated demands, I'd recommend having Whirley do the repairs
herself and withhold $200 from rent.
Sprinkler System
It is unlikely that Whirley will be able to force Spears to repair the sprinkler
system, or to seek any reimbursement from him if she decides to make repairs herself.
While Spears has a duty to keep her rental house from having any untenantable
conditions, and a broken sprinkler system is inconvenient, it does not relate to any of the
untenantable categories enumerated in Franklin Civil Code §541, unless it is indicative or
related to a deeper plumbing issue, and not just the sprinkler box malfunctioning as
Whirley predicts. In fact, the lease also specifies that she must maintain the yard at her
29
own expense, in the Residential Lease Agreement in Section 14(B), so absent statutory
authority, the repair costs will fall onto Whirley as designated under the contract.
Guest Bedroom Sliding Door and Carpet
Whirley may have many options with regards to the guest bedroom sliding door
and carpet, depending on what caused the creation of the half-inch gap between the
bottom of the door and the door frame. As stated above, Spears is required to repair any
subsequent conditions that render the premises untenantable. Franklin Civil Code §540.
A house is deemed untenantable if it lacks "effective waterproofing and weather
protection of roofs and exterior walls, including unbroken windows and doors." Id.
Here, there is a half-inch gap between the door and the door frame of a sliding glass door
that leads to the outside of the house. The gap is allowing outside moisture in the house;
evidence of ineffective waterproofing of an exterior unbroken door. Again, Spears
received written notice from Whirley and failed to repair the door in a reasonable amount
of time. He received a written e-mail about the problem on May 26, and after 2 months,
has made no repairs. Thus, because this is more than 30 days after the Landlord received
notice, Whirley may make the repairs herself. However, because the estimated cost of
the repairs ($1800) is more than her monthly rent ($1200) she will have to sue the
landlord for the cost of repairs, rather than deduct the cost from rent. Franklin Civil Code
§542(a)(2). Again, she could also vacate the premises under Franklin Civil Code
§542(a)(3), and be discharged from further payment of rent, as such statutory authority
would supersede Section 16 of her Residential Lease Agreement claiming that she'd be
responsible for rent payments (Section 11 of her Rental Lease Agreement also gives her
this right - she could argue that her use of the Premises is seriously impaired because she
cannot use the guest room at all, and she could terminate the agreement upon three days'
written notice to Spears) but she would like to remain in the house, so this is not the best
option for her. Additionally, she could withhold some of her rent until Spears makes the
repairs, because she can argue that the condition substantially threatens her health and
safety, because of the mold that has been created. Franklin Civil Code §542(a)(4). The
mold growing around the door and the odor is so problematic that the guest room that
attaches to the door is unusable. In Burk v. Harris, the court noted that the appropriate
reduction in rent may be the "difference between the fair rental value of the premises if
they had been as warranted and the fair rental value as they were during occupancy in
unsafe or unsanitary condition." Burk v. Harris (Fr. Ct. of App. 2002). Under this
formula, she can likely withhold $200 per month, which is the difference the fair rental
value of a 3 bedroom house ($1200) and the fair rental value of the house with one room
rendered unusable- $1000, the average cost of a comparable two bedroom house. The
court may also reduce the tenant's rental obligation by a "percentage corresponding to the
relative reduction of use of the leased premises caused by the landlord's breach," which
wouldn't be as helpful to Whirley, since the guest room that is unusable is actually only
used once or twice a month when family and friends visit her. This would be a smaller
reduction.
30
Additionally, it is important to note that a Landlord has no duty to repair any
untenantable condition that is caused by the Tenant's violation of an affirmative
obligation, if such violation materially contributed to the creation of the untenantable
condition. Franklin Civil code §543. One of a tenant's obligations is not to permit any
person to destroy any part of the dwelling unit. Franklin Civil Code §543(3). If
somehow Spears can show that a guest caused the gap in the door (Whirley isn't sure if
any of the houseguests used the door, and potentially one of them could have put the door
off its track), rather than the door itself being defective, then Spears may have no duty to
repair the door. Additionally, the burden may even be on Whirley to show that the
damage to the door is through the fault of Spears, and not the fault of herself or a guest.
In Shea v. Willowbrook Properties LP (Fr. Ct. of App. 2012), the court found that Shea
could not recover damages where Shea received a bedbug infestation at his rental
apartment, but he had just come back from abroad, and he "failed to demonstrate that the
2011 prolonged bedbug infestation occurred through Willowbrook's fault and through no
fault of his own." Thus, in order to recover damages for the damage caused by the gap in
the door, Whirley will have to show that the gap is not through any fault of her own or
her guests. If, for example, the door is in fact a different size than the door frame, she
will be successfully able to show that's not through the fault of her own.
Repairs to the Walls and Baseboard
Whirley will likely not be able to force Spears to repair the baseboards, and will
not be able to seek reimbursement from Spears for any repairs she does to the baseboards
herself. Again, as noted above, Spears must keep the house tentantable. Whirley could
make an argument that the baseboard destruction is evidence that the dwelling is
untenantable because the "floors" are not "maintained in good repair" under Franklin
Civil Code §541. However, as also noted above, a Landlord has no duty to repair any
untenantable condition that is caused by the Tenant's violation of an affirmative
obligation, if such violation materially contributed to the creation of the untenantable
condition. Franklin Civil Code §543(3) applies to destruction or damage caused by
animals as well as persons, even if their persons or pets are authorized on the premises, as
Whirley's dog is authorized by the Pet Addendum. Because Whirley permitted the dog
Bentley to chew on the baseboard and the walls, her allowance of the dog's destruction
was a significant cause of the damage to the wall and baseboards, and she will not be able
to recover for it.
31
ANSWER TO MPT 2
Franklin Tax Court
Nash v. Franklin Department of Revenue
Post-Hearing Brief [Other sections omitted] III. Legal Argument
1. Under the Internal Revenue Code (IRC) § 162 and Franklin Tax Court case law,
the Nash's are entitled to a deduction for their expenses paid or incurred during
each of the taxable years 2011-2015 in carrying on a trade or business.
The Frank
lin Department of Revenue (FDR) incorrectly denied the Nash's claim
for a deduction for their business expenses. The FDR asserts, wrongly, that the tree-
farming business is not engaged as a "business for profit, due to the lack of a profit
motive". Although, as the Stone case notes, "orders of the [FDR] are presumed correct
and valid", this conclusion by the FDR is clearly erroneous based on the facts, and the
Nash's will meet their burden of proving the error of the FDR's order. Though § 183
disallows deductions for activities not engaged in for profit, under the nine elements of §
1.183-2, the Nash tree-farming operation is a for-profit business, and a going concern that
is on track to turn a profit, and as such is entitled to all applicable business deductions.
The Nash's situation is distinguishable from the Stone case, and the facts in Mr. Nash's
testimony support this. Each factor is examined below.
1. Manner in which the taxpayer carries on the activity.
The Nash's carry on their tree-farming in a business-like manner. They have
placed a sign on the public road each November (see Testimony of Joseph Nash) and
have fixed prices for each of their products. Further, they have set aside a dedicated room
in their house just for the business. The room keeps all business records, catalogues, and
has a computer used for the business. Additionally, the Nash's altered their farming
methods, Mr. Nash took classes on forest management, and spent time on another
Christmas tree farm to gain expertise in new methods to improve profitability. This is
consistent with the IRC section's desire to see an “intent to improve profitability" that
"indicate(s) a profit motive". Contrast this to the Stone case, where the owners did not
know much about horses or their buying, selling, or trading, and made little effort to
improve their profitability. The manner that the Nash's have carried on their enterprise
clearly evidences an intent to become profitable. In just 5 years they have made
substantial progress, as opposed to the Stone case, where 20 years passed without any
progress towards profitability.
2. The expertise of the taxpayer or his advisors.
32
As noted above, Mr. Nash has taken classes on forest management and spent time
on another tree-farm to learn new methods. He also read books on raising Christmas trees
and researched the subject in an orderly way. This clearly indicates a profit motive, and
shows more initiative than in the Stone case, where the Stones had only recreational
experience and only "consulted" with others on some issues not related to profitability.
3. The time and effort expended by the taxpayer in carrying on the activity.
Mrs. Nash has completely left her previous occupation and devoted her full time to
the tree-farm. Additionally, Mr. Nash spends his weekends and summers devoted to the
tree-farm. This is significantly greater than simply "personal or recreational aspects" by
any standard, and Mrs. Nash's retirement from her former occupation especially
evidences their devotion of most of their energies to the tree-farm business. This is
contrasted with the Stone case, where both owners kept full-time jobs separate from their
horse farm. The significant time and effort spent by the Nash's on the tree-farm weighs
strongly in their favor.
4. Expectation that assets used in activity may appreciate in value.
Mr. Nash testified that they intend to make a profit from increased sales and value
produced by the acreage of the farm. The Nash's have invested significantly in new
equipment and methods. This can only create an inference that they intend for the trees
that they farm to generate additional value. They have made significant alterations to the
land, cutting down forest for additional planting space and replanting in organized rows.
This is distinguishable from Stone, as there the Stones stipulated that they had made no
alterations or expected any appreciation.
5. The success of the taxpayer in carrying on other similar or dissimilar activities.
Though the Nash's have not run a successful tree-farm before, that should not
preclude them from doing so now. They have demonstrated an intent and a viable path to
profitability, and this factor should not be held against them.
6. The taxpayer's history of income or losses with respect to the activity.
As the statute notes, a "series of losses during the initial or start-up stage of an
activity may not necessarily be an indication that the activity is not engaged in for profit".
The Nash's have only run their tree-farm for five years, and incurred significant start-up
costs. As an annual business, that relies on a planting, growing, and selling cycle that
takes time, this period is not unreasonably long for a concern to continue without turning
a profit. Additionally, income has steadily increased from 2011-2015, showing that there
is a viable path to profitability. This is contrasted with the Stone case, which saw only
losses over a 20 year period.
33
7. The amount of occasional profits, if any, which are earned.
The tree-farm recently had income of $5,000 for the 2015 year. While there were
large start-up costs, annual costs since then have only been $9,000 - $12,000. Contrasting
to Stone, which the court held never had a path to profit, the Nash's here are well on their
way to profitability.
8. The financial status of the taxpayer.
As noted above, Mrs. Nash is working on the tree-farm full time, and Mr. Nash
expends significant time on the farming activities. Though they have not yet derived
substantial income from the farm, they intend to do so in the future. This is contrasted
with Stone, where both of the Stones had full-time jobs not related to the horse farming.
And, again, the Nash's tree-farm is in the start- up stage, and this factor should not be
held against them.
9. Elements of pleasure or recreation.
As the statute notes, "it is not, however, necessary that an activity be engaged with
the exclusive intention of deriving a profit..." or that the taxpayer has motivations other
than profit or that they derive pleasure from the activity. Here, clearly the Nash's enjoy
tree-farming and gain substantial personal enjoyment from it. The statute does not
require, as the FDR would no doubt prefer, that profitable business activities be dull and
thankless tasks. The Stone case was much more applicable to the FDR's position, Mr.
Stone engaged in rodeos and rode for personal recreational enjoyment. The Nash's case,
while showing pleasure and enjoyment, simply corresponds with individuals who enjoy
what they do and have found a way to earn money doing it.
Based on the foregoing factors, the FDR's order is clearly erroneous in denying the
Nash's deductions for business expenses and should be reversed by this court.
2. Under the Internal Revenue Code § 280A and Franklin Tax Court case law, the
Nash's are entitled to a deduction for their expenses paid or incurred during each of
the taxable years 2011-2015 for the business use of a portion of their home.
The IRC contains a notable exception under § 280A(c) for certain business use,
that allows a deduction for a "portion of the dwelling unit which is exclusively used on a
regular basis". Under Lynn v. FDR, a portion of a dwelling used thusly is judged under
an "all-or-nothing" standard from the earlier case McBride v. FDR. These references to a
report in the Code's legislative history which dictates that "exclusive use of a portion of a
taxpayer's dwelling unit means that the taxpayer must use a specific part of a dwelling
unit solely for the purpose of carrying on his trade or business". And further notes that
34
dual-use for personal and business purposes does not meet that standard. In the Lynn
case, two dwellings were considered. One, which had an area physically separated from
the living area of the home, was held to be for business use and subject to the deduction.
The other, which was a "computer office room" that had both personal and business use,
was not. In the Nash's situation, they have moved all of the bedroom furniture out of the
room in question. In its place, they have placed a desk, chairs, a business computer, and a
TV tuned to the weather channel (consistent with what a tree-farmer would need to know
about weather patterns). The details offered by Mr. Nash in his testimony about the
contents and use of the room clearly indicate that this room is the principal place of
business of the Nash tree-farming operation, and is separate from the rest of the house.
This evidences its business purpose and shows that the FDR's determination that it is not
exclusively for business use is clearly erroneous and should be reversed by this court.
ANSWER TO MPT 2
APPELLATE BRIEF
III. Legal Argument
Under the Internal Revenue Code and the Code of Federal Regulations, in order to
obtain income tax deductions, the activities must be "for profit." The FDR has denied Mr.
and Mrs. Nash full tax deductions for their Christmas Tree Farm because they have
determined that it was an activity not engaged for profit, and therefore they could only
deduct the income which they had made. The FDR also denied their deductions for the
home-office in their residence. The FDR erred on both counts.
A. THE NASH'S CHRISTMAS TREE FARM IS A FOR-PROFIT VENTURE AS
OUTLINED IN THE CODE OF FEDERAL REGULATIONS, AND AS SUCH
THE NASH'S HAVE A RIGHT TO FULL TAX DEDUCTIONS UNDER THE
INTERNAL REVENUE CODE
The CF
R and the court in Stone v. FDR have laid out a nine factor test, which
when balanced make a determination of whether a venture constitute as for profit or as an
activity engaged not for profit. 26 CFR § 1.183
-
2(b)(1)-(9).
1.
THE NASH'S CARRY OUT THEIR ACTIVITIES WITH RESPECT TO THE
CHRISTMAS TREE FARM IN A BUSINESSLIKE MANNER AS EVIDENCED
BY THEIR PURCHASE OF SPECIALIZED EQUIPMENT AND PLANTING OF
ADDITIONAL FIELDS.
35
The fact that a taxpayer is carrying out the activities in a businesslike manner,
maintaining complete and accurate records of the books and records is an indication that
they are a for profit venture. Here, Mr. Nash has testified that they keep all their books
and records in an exclusive home office, further Mr. Nash has shown that they are
carrying out their venture in a businesslike manner by changing their operating methods,
adopting new techniques, and abandoning previous unprofitable methods. They
originated with selling the trees for $15-30 to friends and nearby consumers, and they
have changed that entire operation. They have consulted with businessmen in the field of
Christmas Tree Farming, as well as purchasing specialized equipment to shape and cut
trees, clearing forest areas to plant new trees, and purchasing additional equipment to
faster plant and harvest. They have also hired additional employees to help with the
venture because it has gotten too big to do it on their own.
2. MR. NASH HAS ENGAGED IN EXTENSIVE BUSINESS STUDY OF
CHRISTMAS TREE FARMING THROUGH OBTAINING SPECIALIZED
EDUCATION AND CONDUCTING EXTENSIVE RESEARCH THROUGH
BOOKS AND OTHER CHRISTMAS TREE FARMERS.
In addition to the changes in the business activity, Mr. Nash has done thorough
research and looked to increase his acumen in the business of Christmas Tree Farming.
Mr. Nash has taken classes on forest management, read numerous books on raising trees,
and met a nearby Christmas tree farmer and spent an entire vacation on that farm.
Further, he has kept in touch with that farmer and continued to seek his advice and
business activities and carried out all the advice that the farmer has given to him.
In Stone v. FDR, the court determined that the individuals in their horse farm were
not engaged in the business study because they had no formal educational training, just
recreational training. That is not the case here, Mr. Nash purposefully availed himself to
education in this specific area as it was venture where he sees a profit and potential for
growing and blossoming business.
3. MRS. NASH IS ENGAGED IN THIS VENTURE FULL-TIME WHILE MR.
NASH IS ENGAGED IN THIS VENTURE ON WEEKENDS AND ANY FREE
TIME OUTSIDE OF HIS FULL TIME JOB AT A LOCAL SCHOOL
The CFR and the courts have stated that whether an individual is solely engaged in
the business, and derives no additional income, can show that the taxpayer is engaged in
the business for profit. Here, it is correct that Mr. Nash is a full time associate principal at
a local school; however Mrs. Nash is working full time within the venture. She retired
from her job around the time where the Nash's decided to engage in this venture as a
business, at the same time where they changed their entire business plan and modified it
in a way where they can expect future profits.
36
As such, despite Mr. Nash's full time job, Mrs. Nash's full-time devotion to the
venture as well as Mr. Nash's weekends and free-time devotion shows they intend this to
be a profitable venture. To penalize an individual for having a job and hoping to start a
business is not the manner in which the tax code works, this factor is looking for the
intent of the parties, and despite Mr. Nash's job, there is a clear intent here to be engaged
in a for-profit venture.
In Stone, spending 30 to 40 hours per week on the farm yet still having a full time
job made this factor neutral, therefore it is likely that despite Mr. Nash's job-it is still
clear from Mrs. Nash that this is intended to be a profitable venture.
4. THE NASH'S EXPECT AN APPRECIATION IN VALUE OF ASSETS,
PARTICULARLY OF THE ACERAGE WHICH HAS RECENTLY BEEN
PLANTED THAT THEY ARE WAITING TO BE READY TO SELL TO THEIR
CUSTOMERS
Not only have the Nash's invested in costly equipment (which is the main reason
for their lack of profits) which they expect to appreciate, they have also planted many
additional acres of trees which they expect will appreciate as they continue to grow and
once they are ready to be cut and sold, their income and profits will significantly
increase. They have loyal customers, including some commercial customers, who eagerly
await these baby trees to be ready to be sold.
5. THE FACT THAT THE NASH'S HAVE NEVER HAD ANOTHER BUSINESS
VENTURE BEFORE IS NOT DISPOSITIVE TO WHETHER THEY HAVE A
RIGHT TO FULL TAX DEDUCTIONS UNDER THE IRC
As with all factor tests, there is no one factor which can be dispositive. It is true, as
stated in Mr. Nash's testimony, that they have never had another business venture before,
however that factor in and of itself should not be dispositive in denying their right to full
tax deductions.
6. THE NASH'S HAVE EXPERIENCED MAJOR LOSSES OVER THE PAST
FIVE YEARS, HOWEVER THEY HAVE DECREASED A SIGNIFICANT
AMOUNT AND MR. NASH EXPECTS A PROFIT RETURN SOON
The factor looking at the profits and losses is also not dispositive. While it is true
that the Nash's have never taken a profit, they have seen an increase in income. Further,
the reason that they have not seen a profit is because 2011 was a bad year, the economy
has had a rough time (but it is coming back), and the acreage that they planted is still not
ready. Unlike other business ventures, trees take many years to grow so it is not
unexpected that the first few years of this venture are not profitable. However, the Nash's
37
have been conducting this venture like a business and are holding themselves out as
business people.
The Nash's are easily differentiated from the case of Stone v. FDR, where the
Stone family experienced major losses and the only income they ever experienced was
$4,000 for the sale of a horse. They never had any other income. Here, the Nash's
experience yearly income which has steadily increased, including a $1,500 increase both
from 2013 to 2014, and from 2014 to 2015. It is clear that they are making more and
more money and they can expect future profits.
A taxpayer should not be penalized on their business venture just because they
have not recognized substantial profits which exceed their losses. It is clear here that the
Nash's are working hard to make money and intend to be profitable in their venture.
7. THE NASH'S HAVE EXPERIENCED INCOME, HOWEVER IT HAS GONE
TO RECOVER FROM THEIR LOSSES IN THE PURCHASE OF SPECIALIZED
EQUIPMENT TO MAINTAIN THE FARM
Mr. Nash has stated that the reason they are not experiencing profits is because
they have to pay their debts from the purchase of equipment, including the specialized
equipment, and maintenance on the farm. They also have to pay the newly hired workers
that they are paying salaries to in order to maintain the farm. There has been income, and
as earlier stated, there has been a steady increase in income, but they are backlogged with
a previous expense that was necessary to change this venture into a for profit minded
venture.
Unlike in Stone, where they argued they had purchased expensive race-horse
seamen, here the Nash's have made quality investments in their business venture, which
can be seen by the steady increase in profits.
8. THE FACT THAT MR. AND MRS. NASH DERIVE INCOME FROM MR.
NASH'S FULL TIME JOB AND MRS. NASH'S PENSION IS NOT DISPOSITIVE
OF WHETHER THEY HAVE A RIGHT TO FULL TAX DEDUCTIONS
Like factor 5 and in accordance with the arguments in factor 3, while Mr. Nash
works full time and they experience additional income, it is clear that they intend this to
be the main portion of their lives. The income coming in from
Mr. Nash's full time job and Mrs. Nash's pension should not penalize them, it should not
be wrong for them to have money to live on while they are working to get their business
venture off the ground.
It is true that neither Mr. nor Mrs. Nash currently take a salary, and while that may
be equated as the same as in Stone, it is very different. It has only been five years, and
38
2011 as conceded by Mr. Nash was a very bad year for the venture. In Stone, they never
took a salary for 20 years. That is four times as long. They are not equivalent at all. For
the Nash's to take a profit would harm the venture and it is clear that they hope for it to
continue growing and to be profitable in the future.
9. WHILE MR. NASH HAS STATED HE ENJOYS CHRISTMAS TREE
FARMING, THE NASH'S SET OUT IN THIS VENTURE WITH THE INTENT
TO MAKE MONEY, IT WAS NOT A PRE-EXISTING RECREATIONAL
ACTIVITY
In Stone, the court looked to the fact that Mr. Stone had been riding horses since
he was a child, and the fact that the Stones only went to rodeo and show events, and did
not have a plan which looked like anything more than a hobby to them. The fact that Mr.
Nash stated that he enjoys tree farming and is fascinated by it does not show that this is a
hobby. Mr. Nash specifically stated that he and his wife saw the potential and that is
when he became interested in it, by taking classes and reading books with the very intent
to run this business. Mr. Nash's activities clearly show an intent for this venture to be
profitable, and as such it is not a hobby or recreational. It is not dis-allowed to enjoy
something you are doing while you are doing it for profit, the IRC does not require
misery to equate with profit, Mr. Nash can enjoy his work in a for profit venture, which
this is.
As such, the Nash's should be deemed to be a for-profit venture, and are therefore
entitled to full tax deductions. The factors (1) (2) (3) (4) (6) (7) and (9) show a clear
intent to exist for profit. The CFR lays this out as a balancing test, and that is what it is.
The factors in favor of the Nash’s being engaged in a for profit venture far outweigh the
5th and 8th factors that look to the fact that Mr. Nash has a job and Mrs. Nash has a
pension, it is clear that the Nash's have an intent to retain a profit, but unfortunately have
yet to
B. THE NASH'S HAVE A RIGHT TO TAX DEDUCTIONS FOR THE HOME-
OFFICE IN THEIR RESIDENCE AS IT IS USED SOLELY AS A PRINCIPAL
PLACE OF BUSINESS FOR THEIR CHRISTMAS TREE FARM AS
EVIDENCED BY A PHYSICAL CONVERSION FROM RESIDENCE TO
HOME-OFFICE
The Internal Revenue Code and Code of Federal Regulations state that generally,
there is no allowed deduction with respect to a personal residence of a taxpayer.
However, there is an exception for expenses allocated to a portion of that residence which
is used exclusively and on a regular basis as the principal place of business for any trade
or business of the taxpayer. This exception is outlined in the case of Lynn v. Franklin
Department of Revenue, where the case looks to legislative history and the case of
McBride v. FDR to lay out the "exclusive use" requirement and test. It states that the
39
portion being claimed for a tax deduction must be used solely for the purpose of carrying
on the trade or business. In Lynn v. FDR, the court determined that the first residence,
where the plaintiff made (1) a physical separation from living areas; and (2) a physical
conversion from residence to office; and (3) had a separate entrance constituted as an
exclusive use under the test and therefore was allowed to gain the full tax deductions
under the exception. However, in that same case, the court determined that a room in his
apartment was not an exclusive use because the plaintiff failed to offer details about what
is in the "exclusive" area and how the room was used. Further, the court said that the
presence of a television coupled with the lack of testimony about the business use, as well
as the conclusion that the computer was likely used for both personal and business
purposes constituted it as not-exclusive and therefore did not obtain the full tax
deductions.
Here, Mr. Nash has testified that they have a room which they physically
converted from a bedroom (before forming this business venture) into a home-office.
They took out the bed and have a desk and two chairs in the office, as well as a television
which is used to watch the weather channel for business purposes of growing trees.
Additionally, Mr. Nash testified that they keep records, catalogues, and books that they
consult in the room and that the computer in the office is for business use, and nothing
else. He affirmatively stated in his testimony that nothing happens there but business.
As such, based on Mr. Nash's testimony and the fact that there was physical
conversion (which the court in Lynn stated is a factor in identifying an exclusive use) the
home-office fulfills the exclusive use test and therefore the Nash's should be able to
obtain tax deductions for their home office under the exception.